Re: [R] survival survfit with newdata

2012-05-17 Thread Damjan Krstajic

Thanks David for prompt reply. I agree with you. However, I still fail to get 
the survfit function to work with newdata. In my previous example I changed the 
column names of testX matrix and I still fail. 

 colnames(testX)-names(coxph.model$coefficients)
 sfit- survfit(coxph.model,newdata=data.frame(testX))
Error in model.frame.default(formula = Surv(trainTime, trainStatus) ~  :
  variable lengths differ (found for 'trainX')

What would be solution in my simple example to get the survival curves for 
testX? Thanks in advance. DK

 CC: r-help@r-project.org
 From: dwinsem...@comcast.net
 To: dkrsta...@hotmail.com
 Subject: Re: [R] survival survfit with newdata
 Date: Thu, 17 May 2012 00:52:55 -0400
 
 
 On May 16, 2012, at 5:08 PM, Damjan Krstajic wrote:
 
 
  Dear all,
 
  I am confused with the behaviour of survfit with newdata option.
 
 Yes. It has the same behavior as any other newdata/predict from  
 regression. You need to supply a dataframe with the same names as in  
 the original formula. Doesn't look as though that strategy is being  
 followed. The name of the column needs to be 'trainX' since that was  
 what was its name on the RHS of hte formula,  and you may want to  
 specify times. If you fail to follow those rules, the function falls  
 back on offering estimates from the original data.
 
 
  I am using the latest version R-2-15-0. In the simple example below  
  I am building a coxph model on 90 patients and trying to predict 10  
  patients. Unfortunately the survival curve at the end is for 90  
  patients.
 
 As is proper with a malformed newdata argument.
 
  Could somebody please from the survival package confirm that this  
  behaviour is as expected or not - because I cannot find a way of  
  using 'newdata' with really new data. Thanks in advance. DK
 
  x-matrix(rnorm(100*20),100,20)
 
 
  time-runif(100,min=0,max=7)
 
 
  status-sample(c(0,1), 100, replace = TRUE)
  trainX-x[11:100,]
 
  trainTime-time[11:100]
 
  trainStatus-status[11:100]
 
  testX-x[1:10,]
  coxph.model-
  coxph(Surv(trainTime,trainStatus)~ trainX)
  sfit- survfit(coxph.model,newdata=data.frame(testX))
 
 
  dim(sfit$surv)
 
  [1] 90 90
 
 
  
  [[alternative HTML version deleted]]
 
  __
  R-help@r-project.org mailing list
  https://stat.ethz.ch/mailman/listinfo/r-help
  PLEASE do read the posting guide http://www.R-project.org/posting-guide.html
  and provide commented, minimal, self-contained, reproducible code.
 
 David Winsemius, MD
 West Hartford, CT
 
  
__
R-help@r-project.org mailing list
https://stat.ethz.ch/mailman/listinfo/r-help
PLEASE do read the posting guide http://www.R-project.org/posting-guide.html
and provide commented, minimal, self-contained, reproducible code.


[R] hu6800cdf

2012-05-17 Thread srod
Hi,

I'm using a command in bioconductor that seems to require a package called
hu6800cdf. I've installed this properly but I still get the same error:

 Could not find array definition file ' hu6800cdf.qcdef '. Simpleaffy does
not know the QC parameters for this array type.
See the package vignette for details about how to specify QC parameters
manually.

I've tried specifying the cdfname file by using

data-ReadAffy(cdfname=hu6800cdf)  but it still returns the same error. 

The version of R that I'm using is 2.15 and the bioconductor version is
2.10.

Does anyone know how to solve this?

Thanks,

srod

--
View this message in context: 
http://r.789695.n4.nabble.com/hu6800cdf-tp4630337.html
Sent from the R help mailing list archive at Nabble.com.

__
R-help@r-project.org mailing list
https://stat.ethz.ch/mailman/listinfo/r-help
PLEASE do read the posting guide http://www.R-project.org/posting-guide.html
and provide commented, minimal, self-contained, reproducible code.


Re: [R] Error on easy way for JoSAE Package

2012-05-17 Thread ana24maria
I have attached to this message the first 20 lines of the output for
dput(amigo). Unfortunately, i can't send it the overall output.
Thank you.

On Wed, May 16, 2012 at 6:10 PM, David Winsemius [via R] 
ml-node+s789695n4630253...@n4.nabble.com wrote:


 On May 16, 2012, at 1:33 AM, ana24maria wrote:

  Thank you very much.
  After using dput and the easy way ( result -
  eblup.mse.f.wrap(domain.data
  = amigo, lme.obj = fit.lme)),
  i have got the following error:
 
  Error in `[.data.frame`(sample.data, , variabs) :
   undefined columns selected

 What John was asking you to do was at your console just type:

 dput(amigo)

 ... and then copy the output to an email and send that to the list.
 Your first posting had data that was ambiguous as to content as well
 as mangled by the various email clients and servers that processed on
 the path to our eyes.

 
  What should I do?

 You should also read the Posting Guide.

  --
  View this message in context:
 http://r.789695.n4.nabble.com/Error-on-easy-way-for-JoSAE-Package-tp4625684p4630220.html

 

  PLEASE do read the posting guide
 http://www.R-project.org/posting-guide.html
  and provide commented, minimal, self-contained, reproducible code.
 --

 David Winsemius, MD
 West Hartford, CT

 __
 [hidden email] http://user/SendEmail.jtp?type=nodenode=4630253i=0mailing 
 list
 https://stat.ethz.ch/mailman/listinfo/r-help
 PLEASE do read the posting guide
 http://www.R-project.org/posting-guide.html
 and provide commented, minimal, self-contained, reproducible code.


 --
  If you reply to this email, your message will be added to the discussion
 below:

 http://r.789695.n4.nabble.com/Error-on-easy-way-for-JoSAE-Package-tp4625684p4630253.html
  To unsubscribe from Error on easy way for JoSAE Package, click 
 herehttp://r.789695.n4.nabble.com/template/NamlServlet.jtp?macro=unsubscribe_by_codenode=4625684code=YW5hMjRtYXJpYUBnbWFpbC5jb218NDYyNTY4NHwxNzE3MDQ5Njc1
 .
 NAMLhttp://r.789695.n4.nabble.com/template/NamlServlet.jtp?macro=macro_viewerid=instant_html%21nabble%3Aemail.namlbase=nabble.naml.namespaces.BasicNamespace-nabble.view.web.template.NabbleNamespace-nabble.view.web.template.NodeNamespacebreadcrumbs=notify_subscribers%21nabble%3Aemail.naml-instant_emails%21nabble%3Aemail.naml-send_instant_email%21nabble%3Aemail.naml



--
View this message in context: 
http://r.789695.n4.nabble.com/Error-on-easy-way-for-JoSAE-Package-tp4625684p4630339.html
Sent from the R help mailing list archive at Nabble.com.
[[alternative HTML version deleted]]

__
R-help@r-project.org mailing list
https://stat.ethz.ch/mailman/listinfo/r-help
PLEASE do read the posting guide http://www.R-project.org/posting-guide.html
and provide commented, minimal, self-contained, reproducible code.


[R] Job opportunity in Beijing, China at Xian-Janssen Pharmaceutical Ltd

2012-05-17 Thread Fei Chen

The Quantitative Decision Strategies group at Janssen Research  Development, 
Johnson  Johnson, is looking for a candidate to represent QDS in Beijing, 
China in the subsidiary company of  Xian-Janssen Pharmacetical Ltd. The basic 
requirements for this candidate are 1) 3+ years experience in a quantitative 
field, but not necessarily pharmaceutical; 2) PhD in statistics or related 
field preferred but not absolutely necessary; 3) speak fluent Chinese and 
English but willing to be located in Beijing; 4) excellent communications 
skill. The full job description follows, if interested, please send resume 
directly to 

Liwei Wang [lwan...@its.jnj.com]

https://jnjc.taleo.net/careersection/2/jobdetail.ftl?lang=enjob=03XM



Senior Quantitative Scientist-03XM

Description
 
PRINCIPAL RESPONSIBILITIES: List major responsibilities and duties of the 
position. List most complex or difficult parts of job first and indicate 
percentage of time required to perform each task. Describe those quantitative 
aspects of the position which reflect measures that are applicable to the 
position's major responsibilities or end results. (TYPE BELOW THIS LINE)
•   Support for broad implementation of innovative approaches: Implement 
strategy for promoting the broader utilization of innovative approaches across 
the development portfolio, through coordinated efforts involving clinical, 
operational, regulatory areas. Bring together key stakeholders from different 
key areas, identify potential hurdles for broader use of innovation in drug 
development and participate in finding solutions for them. Collaborate in 
talent identification for QDS group, hiring and development. Ensure focus on 
innovation is maintained in QDS group.
 
•   Evaluate and implement innovative approaches: participate in hands-on 
implementation of innovative approaches, through direct interactions with 
clinical teams and other relevant groups. This involves, but is not limited to 
planning, running, and summarizing simulation studies to evaluate the operating 
characteristics of alternative designs and methods; and participating in the 
implementation of the selected approaches by engaging in protocol and SAP 
write-up. Engage external consultants and qualified CROs to provide support in 
the development and implementation of innovative methods, supervising them as 
needed.
 
•   Identification of opportunities for innovation: participate in CDP 
reviews/discussions; interact directly with clinical teams and therapeutic area 
representatives for early identification of opportunities for innovative 
approaches. Assess feasibility and help establish prioritization of identified 
opportunities. Ensure implementation of selected opportunities
 
 
•   Ensuring proper knowledge and awareness of innovative methods and 
designs: help identify, develop, organize, provide and/or arrange training and 
presentations on innovative approaches across the Development organization 
(including statisticians, modelers, clinicians, and other stakeholder groups) 
to ensure awareness and adequate knowledge about these methods. Keep up-to-date 
knowledge of designs and analysis methods for clinical trials, including 
adaptive designs, model-based methods, etc.
 
•   External collaborations, visibility, and impact: engage in external 
collaborations within professional associations, participating  in working 
groups, organizing sessions and presenting at scientific meetings, and 
publishing in peer reviewed journals. Influence the pharmaceutical industry and 
regulatory environments, by engaging in scientific advocacy working groups and 
committees.
 

Qualifications
 
 
•   Doctorate (Ph.D.) in statistics or related field, with a minimum of 3 
years experience in drug development and/or applied quantitative methods for 
decision making, significant knowledge of scientific programming
•   Familiarity with industry principles of drug development, modeling and 
simulation, clinical pharmacology, clinical trial design and regulatory 
guidelines.
•   Sound knowledge of biostatistics applied to clinical trials and 
model-based drug development.
•   Working knowledge of statistical software, such as SAS, R, and S-PLUS, 
WinBugs
•   Have experience with trial simulations to design and document adaptive 
trials and have working knowledge of Bayesian methods.  Stay current with the 
latest statistical methodologies with focus on trial optimization, simulation 
and adaptive designs.
•   Good verbal and written communication skills, including formal 
presentation skills.  Experience presenting to technical and lay groups at 
public meetings desirable.  Written skills as evidenced by publication and 
journal articles also desirable.
•   Ability to successfully multi-task and work independently, under 
minimal supervision; excellent teamwork skills.
•   Ability to influence, 

[R] step function stops with subscript out of bounds

2012-05-17 Thread David A Vavra
I've been having a problem using the step function to evaluate models. I've
simplified the code and get the same problem using the dataset Titanic. The
relevant code and output is below. The problem disappears (i.e., 'step' runs
correctly) if I rerun the code but change the 'loglm' call to explicitly
reference Titanic instead of X (as in: loglm(as.formula(Y),data=Titanic)).

What is causing this?

TIA,
DAV 

--

 catn-function(...) cat(...,\n)
 local({  X-Titanic; print(class(X)); 
   Y-paste('~',paste(names(dimnames(X)),collapse=*)); 
   print(Y); 
   sm-loglm(as.formula(Y),data=X); 
   catn(SM); print(sm); catn('running'); 
   step(sm,direction='backward')  })

Output:

[1] table
[1] ~ Class*Sex*Age*Survived
SM 
Call:
loglm(formula = as.formula(Y), data = X)

Statistics:
 X^2 df P( X^2)
Likelihood Ratio   0  01
Pearson  NaN  01
running 
Start:  AIC=64
~Class * Sex * Age * Survived

Error in loglin(data, margins, start = start, fit = fitted, param = param,
: 
  subscript out of bounds

Enter a frame number, or 0 to exit   

1: local({
X - Titanic
print(class(X))
Y - paste(~, paste(names(dimnames(X)), collapse = *))
print(Y)
sm - loglm(as.formula(Y), data = X
 2: eval.parent(substitute(eval(quote(expr), envir)))
 3: eval(expr, p)
 4: eval(expr, envir, enclos)
 5: eval(quote({
X - Titanic
print(class(X))
Y - paste(~, paste(names(dimnames(X)), collapse = *))
print(Y)
sm - loglm(as.formula(Y), dat
 6: eval(expr, envir, enclos)
 7: #1: step(sm, direction = backward)
 8: #1: drop1(fit, scope$drop, scale = scale, trace = trace, k = k, ...)
 9: #1: drop1.default(fit, scope$drop, scale = scale, trace = trace, k = k,
...)
10: #1: update(object, as.formula(paste(~ . -, tt)), evaluate = FALSE)
11: #1: update.loglm(object, as.formula(paste(~ . -, tt)), evaluate =
FALSE)
12: #1: eval.parent(call)
13: #1: eval(expr, p)
14: #1: eval(expr, envir, enclos)
15: #1: loglm(formula = ~Class + Sex + Age + Survived + Class:Sex +
Class:Age + Sex:Age + Class:Survived + Sex:Survived + Age:Survived +
Class:Sex:Age + Class:
16: #1: loglm1(formula, data, ..., .call = .call, .formula = .formula)
17: #1: loglm1.default(formula, data, ..., .call = .call, .formula =
.formula)
18: #1: loglin(data, margins, start = start, fit = fitted, param = param,
eps = eps, iter = iter, print = print)

Selection: 0

__
R-help@r-project.org mailing list
https://stat.ethz.ch/mailman/listinfo/r-help
PLEASE do read the posting guide http://www.R-project.org/posting-guide.html
and provide commented, minimal, self-contained, reproducible code.


[R] oldlogspline probabilities

2012-05-17 Thread Beutel, Terry S
I using oldlogspline (from logspline package) to model data distributions, and 
having a problem.

My data are search area sizes. They are based on circular search radii from 
random points to the nearest edge of the nearest grass tussock. Search area 
sizes are distributed from 0 (the random point intercepts a tussock) and 
upwards (as points are further from any tussocks). The density of all my 
distributions (using doldlogspline) are highest at size=0, and decline as 
search area increases.

I am most interested in the probability of a value in the distribution being 
equal to 0 (ie probability of a direct hit on a tussock). I know I can just use 
the proportion of actual hits, but am curious to compare this to an estimate 
from a density estimation. Unfortunately when using poldlogspline, this 
probability is always=0 (simulated data example is below). How can this be, 
given that the density is highest at area=0?

 simdat-c(rep(0,8),rexp(92))
 myspline-oldlogspline(simdat,lbound=0)
 poldlogspline(fit=myspline, q=0)
[1] 0

Any help to work out the probability of an area value in my distribution = 0 
would be appreciated


Terry Beutel

Agri-Science Queensland

[[alternative HTML version deleted]]

__
R-help@r-project.org mailing list
https://stat.ethz.ch/mailman/listinfo/r-help
PLEASE do read the posting guide http://www.R-project.org/posting-guide.html
and provide commented, minimal, self-contained, reproducible code.


[R] trouble with multi statement queries

2012-05-17 Thread tensegrity
Using both RMySQL or RODBC packages  I can connect to a mySQL database and
return results from simple queries.  However, I cannot return anything from
queries even with just two lines, each ending with semicolons.  Using
RMySQL, the latter yields a message about an error in mysqlFetch, RS-DBI
driver, corrupt resultSet, missing fieldDescription.  Using RMySQL I have
set the client.flag=CLIENT_MULTI_STATEMENTS but to no avail.  I am running
Debian Squeeze.  Please help


--
View this message in context: 
http://r.789695.n4.nabble.com/trouble-with-multi-statement-queries-tp4630341.html
Sent from the R help mailing list archive at Nabble.com.

__
R-help@r-project.org mailing list
https://stat.ethz.ch/mailman/listinfo/r-help
PLEASE do read the posting guide http://www.R-project.org/posting-guide.html
and provide commented, minimal, self-contained, reproducible code.


Re: [R] How to sum and group data by DATE in data frame

2012-05-17 Thread Cren
Thank you, Michael :)

Michael Weylandt wrote
 
 If that doesn't nail it down, I'll need you to answer the questions I
 asked in my previous email.
Previously I made a mistake with *dput()*, this is the correct output:

 dput(X)
new(timeSeries
, .Data = structure(c(124.3, 124.38, 124.67, 125.19, 124.9, 125.27,
125.5, 
125.58, 125.91, 125.8, 125.83, 126.215, 126.25, 126.25, 124.901, 
124.43, 124.4654, 124.46, 124.68, 124.86, 124.73, 125.22, 125.48, 
125.5601, 125.4091, 125.15, 125.43, 125.481, 125.91, 125.29, 
124.79, 124.77, 124.7, 124.37, 124.56, 124.86, 125.3, 125.59, 
125.95, 125.73, 126.27, 126.26, 127.33, 126.37, 126.46, 126, 
126.06, 126.2662, 126.23, 126.4499, 127.12, 127.48, 127.49, 127.69, 
127.88, 127.88, 124.51, 124.42, 124.92, 125.18, 125.23, 124.81, 
125.07, 124.61, 123.8869, 123.24, 123.3329, 123.6, 123.19, 123.161, 
123.96, 123.58, 123.42, 123.68, 124.19, 123.985, 124.24, 124.61, 
124.6566, 124.18, 123.795, 124.36, 124.32, 124.32, 126.1101, 
126.42, 126.8181, 126.79, 126.675, 126.68, 126.685, 126.725, 
126.65, 126.77, 126.55, 126.63, 126.67, 126.66, 125.8829, 126.05, 
125.97, 125.99, 125.59, 125.21, 125.31, 125.46, 125.39, 125.23, 
124.93, 125.32, 125.46, 125.46, 125.62, 125.34, 125.163, 125.01, 
125.115, 125.67, 126.08, 126.15, 126.33, 126.19, 126.3955, 126.71, 
126.083, 126.083, 125.23, 124.96, 125.595, 125.77, 125.4, 125.38, 
125.54, 126.01, 126.05, 126.01, 125.72, 124.79, 124.05, 124.08, 
123.35, 123.85, 123.75, 123.5, 123.58, 122.9285, 122.14, 122.02, 
121.86, 121.58, 121.79, 121.68, 122.18, 122.105, 122.37, 122.02, 
122.4765, 122.31, 121.99, 121.8401, 121.85, 122.17, 122.27, 122.26, 
122.08, 122.02, 121.979, 121.979, 119.74, 119.55, 118.95, 119.12, 
119.14, 118.91, 119.0201, 119.235, 118.93, 119.51, 119.51, 120.04, 
119.63, 119.661, 119.77, 119.78, 118.89, 118.85, 118.9, 119.2671, 
119.73, 119.8, 119.52, 119.43, 119.16, 119.28, 119.19, 119.19, 
117.43, 117.18, 117.3, 116.87, 117.07, 117.0565, 117.4497, 117.47, 
117.07, 116.856, 116.91, 117.075, 116.6, 116.56, 117.09, 117.361, 
117.35, 117.2787, 116.72, 116.76, 116.34, 116.34, 116.34, 119.88, 
119.98, 119.8095, 119.995, 119.88, 119.685, 119.48, 119.845, 
119.505, 119.65, 119.36, 119.02, 119.73, 119.71, 120.44, 120.65, 
120.45, 120.53, 120.41, 120.67, 120.399, 120.39, 119.91, 120.069, 
120.18, 120.37, 120.05, 120.05, 123.9711, 124.01, 124.47, 124.14, 
124.14, 124.12, 124.14, 123.94, 123.86, 124, 124.07, 124.33, 
125.04, 124.99, 125.47, 125.39, 124.91, 124.5802, 124.88, 124.89, 
124.48, 124.88, 125.335, 125.12, 125.07, 125.25, 124.96, 124.97, 
126.02, 126.41, 126.19, 125.93, 125.5199, 125.575, 125.6702, 
125.471, 125.506, 125.41, 125.14, 125.18, 124.86, 124.86, 126.63, 
127.01, 126.881, 126.86, 126.81, 126.86, 127.05, 126.945, 126.2946, 
126.1511, 126.28, 125.54, 126.24, 126.22, 125.99, 126.41, 126.12, 
126.05, 126.17, 126.13, 126.14, 126.43, 126.26, 126.35, 126.89, 
126.645, 126.26, 126.26, 125.5, 125.12, 125.49, 125.86, 125.96, 
126.28, 126.37, 126.35, 126.115, 126.1, 125.83, 126.42, 126.73, 
126.73, 125.79, 125.32, 125.4, 124.98, 125.0601, 125, 124.84, 
124.5, 124.6246, 124.56, 124.29, 124.78, 123.98, 123.95, 125.13, 
125.24, 125.48, 125.84, 125.73, 125.76, 125.87, 126.02, 125.9, 
126.09, 126.19, 126.21, 126.05, 126.05, 124.208, 123.96, 124.04, 
123.9, 123.4, 123.48, 123.5046, 123.55, 123.5911, 123.59, 123.71, 
123.825, 124.19, 124.21, 125.38, 125.0199, 124.64, 124.64, 124.85, 
124.675, 124.79, 124.67, 124.79, 124.53, 123.86, 123.1, 123.05, 
123.05, 122.42, 122.25, 121.81, 121.77, 121.66, 121.94, 122.1484, 
122.5, 122.0654, 122.07, 121.67, 121.96, 121.71, 121.74, 123.06, 
122.57, 122.21, 122.55, 122.31, 122.71, 122.47, 122.4773, 122.405, 
122.31, 122.18, 122.37, 122.18, 122.185, 122.44, 122.68, 122.69, 
122.51, 121.9, 121.74, 121.6, 121.78, 121.5999, 121.46, 121.67, 
121.57, 121.59, 121.59, 122.05, 121.892, 121.47, 121.28, 120.85, 
121.09, 121.15, 121.02, 121.24, 121.02, 121.07, 120.21, 120.32, 
120.29, 122.99, 123.3, 123.35, 123.47, 123.4254, 123.469, 123.54, 
123.72, 123.7, 123.72, 123.9, 123.97, 123.92, 123.93, 123.26, 
123.49, 123.31, 123.13, 123.2, 122.85, 123.34, 123.34, 123.59, 
123.57, 123.73, 124.03, 124.17, 124.17, 124.98, 124.97, 124.78, 
124.8, 124.88, 125.25, 125.22, 125.3, 125.201, 125.3289, 125.1, 
125.345, 125.4, 125.27, 125.66, 125.68, 125.73, 125.946, 126.04, 
126.11, 126.2, 126.08, 126.11, 126.02, 126.05, 126.005, 126.41, 
126.39, 126.77, 126.78, 126.39, 126.57, 126.55, 126.635, 126.58, 
126.55, 126.72, 126.62, 126.7, 126.65, 126.49, 126.49, 125.74, 
125.4, 125.39, 125.1, 125.05, 125.08, 124.94, 125.14, 125.29, 
125.13, 124.99, 124.78, 124.88, 124.83, 125.42, 125.6991, 125.83, 
125.64, 125.9505, 125.8001, 125.83, 125.8077, 125.88, 125.89, 
125.86, 126, 126.12, 126.12, 126.1685, 126.08, 126.11, 126.22, 
126.03, 126, 126.04, 126, 126.04, 125.78, 125.839, 125.94, 125.5, 
125.5, 127.95, 128.19, 128.24, 128.18, 127.955, 127.937, 127.52, 
127.56, 127.73, 127.85, 128.04, 127.78, 127.49, 127.495, 127.36, 
126.9246, 

[R] Error in lasso regression (linear dependence issue?)

2012-05-17 Thread YaoPau
I have a rather large dataset which I just inserted hundreds of interaction
terms, and I guess R is saying there is a linear dependence when I run: 
l1ce(y~., data=x, bound = .5, absolute.t = FALSE).

The error is below:

/Error in if (any(bound  0)) stop('bound'(s) must be non negative) : 
  missing value where TRUE/FALSE needed
In addition: Warning message:
In l1ce(poss6 ~ ., data = x, bound = 0.5, absolute.t = FALSE) :
  X Matrix (transformed variables) has rank 1081  p = 1086, i.e., is
deficient/

Google doesn't show anything regarding the first error.  Any ideas what is
causing this?

Also, how can I find which column vectors have a linear dependence?

--
View this message in context: 
http://r.789695.n4.nabble.com/Error-in-lasso-regression-linear-dependence-issue-tp4630347.html
Sent from the R help mailing list archive at Nabble.com.

__
R-help@r-project.org mailing list
https://stat.ethz.ch/mailman/listinfo/r-help
PLEASE do read the posting guide http://www.R-project.org/posting-guide.html
and provide commented, minimal, self-contained, reproducible code.


Re: [R] step function stops with subscript out of bounds

2012-05-17 Thread Prof Brian Ripley

On 17/05/2012 09:25, David A Vavra wrote:

I've been having a problem using the step function to evaluate models. I've
simplified the code and get the same problem using the dataset Titanic. The
relevant code and output is below. The problem disappears (i.e., 'step' runs
correctly) if I rerun the code but change the 'loglm' call to explicitly
reference Titanic instead of X (as in: loglm(as.formula(Y),data=Titanic)).

What is causing this?


A lack of understanding of 'non-standard evaluation'.  X (or at least, 
the X you want) is not visible from the standard search path.




TIA,
DAV 

--


catn-function(...) cat(...,\n)
local({  X-Titanic; print(class(X));

Y-paste('~',paste(names(dimnames(X)),collapse=*));
print(Y);
sm-loglm(as.formula(Y),data=X);
catn(SM); print(sm); catn('running');
step(sm,direction='backward')  })


Which will tell you
Error in eval(expr, envir, enclos) : could not find function loglm

If you correct that and use a vanilla session you will get

Error in loglm(formula = ~Class + Sex + Age + Survived + Class:Sex + 
Class:Age +  :

  object 'X' not found

which is more informative.

So the solution is to

- use less easily masked names than 'X'.
- ensure the data object is visible on the search path.



Output:

[1] table
[1] ~ Class*Sex*Age*Survived
SM
Call:
loglm(formula = as.formula(Y), data = X)

Statistics:
  X^2 df P(  X^2)
Likelihood Ratio   0  01
Pearson  NaN  01
running
Start:  AIC=64
~Class * Sex * Age * Survived

Error in loglin(data, margins, start = start, fit = fitted, param = param,
:
   subscript out of bounds

Enter a frame number, or 0 to exit

1: local({
 X- Titanic
 print(class(X))
 Y- paste(~, paste(names(dimnames(X)), collapse = *))
 print(Y)
 sm- loglm(as.formula(Y), data = X
  2: eval.parent(substitute(eval(quote(expr), envir)))
  3: eval(expr, p)
  4: eval(expr, envir, enclos)
  5: eval(quote({
 X- Titanic
 print(class(X))
 Y- paste(~, paste(names(dimnames(X)), collapse = *))
 print(Y)
 sm- loglm(as.formula(Y), dat
  6: eval(expr, envir, enclos)
  7: #1: step(sm, direction = backward)
  8: #1: drop1(fit, scope$drop, scale = scale, trace = trace, k = k, ...)
  9: #1: drop1.default(fit, scope$drop, scale = scale, trace = trace, k = k,
...)
10: #1: update(object, as.formula(paste(~ . -, tt)), evaluate = FALSE)
11: #1: update.loglm(object, as.formula(paste(~ . -, tt)), evaluate =
FALSE)
12: #1: eval.parent(call)
13: #1: eval(expr, p)
14: #1: eval(expr, envir, enclos)
15: #1: loglm(formula = ~Class + Sex + Age + Survived + Class:Sex +
Class:Age + Sex:Age + Class:Survived + Sex:Survived + Age:Survived +
Class:Sex:Age + Class:
16: #1: loglm1(formula, data, ..., .call = .call, .formula = .formula)
17: #1: loglm1.default(formula, data, ..., .call = .call, .formula =
.formula)
18: #1: loglin(data, margins, start = start, fit = fitted, param = param,
eps = eps, iter = iter, print = print)

Selection: 0

__
R-help@r-project.org mailing list
https://stat.ethz.ch/mailman/listinfo/r-help
PLEASE do read the posting guide http://www.R-project.org/posting-guide.html
and provide commented, minimal, self-contained, reproducible code.



--
Brian D. Ripley,  rip...@stats.ox.ac.uk
Professor of Applied Statistics,  http://www.stats.ox.ac.uk/~ripley/
University of Oxford, Tel:  +44 1865 272861 (self)
1 South Parks Road, +44 1865 272866 (PA)
Oxford OX1 3TG, UKFax:  +44 1865 272595

__
R-help@r-project.org mailing list
https://stat.ethz.ch/mailman/listinfo/r-help
PLEASE do read the posting guide http://www.R-project.org/posting-guide.html
and provide commented, minimal, self-contained, reproducible code.


[R] Complex sort problem

2012-05-17 Thread Axel Urbiz
Dear List,

Is there a way I can sort a sample based on a sort index constructed from
the data from which the sample is taken? Basically, I need to take 'many'
samples from the same source data and sort them. This can be very time
consuming for long vectors. Is there any way I can sort the data only once
initially, and use that sort order for the samples?

I believe that idea is what is implemented in tree-based classifiers, so
the data is sorted only once initially and that sort order is used for the
child nodes.


set.seed(12345)
x - sample(0:100, 10)
x.order - order(x)
x.sorted - x[x.order]

sample.ind - sample(1:length(x), 5, replace = TRUE)  #sample 1/2 size with
replacement
x.sample - x[sample.ind]

x.sample.sorted -   #??? (without sorting again)

Thanks for any help on this.

Regards,
Axel.

[[alternative HTML version deleted]]

__
R-help@r-project.org mailing list
https://stat.ethz.ch/mailman/listinfo/r-help
PLEASE do read the posting guide http://www.R-project.org/posting-guide.html
and provide commented, minimal, self-contained, reproducible code.


Re: [R] how disable the Error massage in read.table() no lines available in input

2012-05-17 Thread Rui Barradas

Hello,

Apparently this is a follow-up from  an earlier post.
I had answered but a misplaced comma in the subject line started another 
thread, that you haven't read.

My original answer is in

http://r.789695.n4.nabble.com/Re-Problem-to-resolve-a-step-for-reading-a-large-TXT-and-split-in-several-file-td4630242.html

As for your question, see the error trapping functions help pages.

?try
?tryCatch

And, in readLines the correct option would be ok = TRUE, warn = TRUE is 
about an end of text marker.


?readLines

Hope this helps,

Rui Barradas

Em 17-05-2012 11:00, gianni lavaredo escreveu:

Date: Wed, 16 May 2012 19:06:14 +0200
From: gianni lavaredogianni.lavar...@gmail.com
To:r-help@r-project.org
Subject: [R] how disable the Error massage in read.table()  no lines
available in input
Message-ID:
CAJ6JbR-H6p9A+hZk-exGYDcPdC__SLBLrM_oRw-=j+du3yg...@mail.gmail.com
Content-Type: text/plain

Dear Researchers,

I am looking a way to disable the Error massage in read.table() as warn =
TRUE in readLines(), when the lines are empty

Error in read.table(con, header = F, sep =  , nrow = n) :
   no lines available in input

thanks for all suggestions
Gianni

[[alternative HTML version deleted]]



__
R-help@r-project.org mailing list
https://stat.ethz.ch/mailman/listinfo/r-help
PLEASE do read the posting guide http://www.R-project.org/posting-guide.html
and provide commented, minimal, self-contained, reproducible code.


[R] Is there R 2.4 version_

2012-05-17 Thread Alaios
Dear all,
I am trying to install the rJava package.
I am getting the following message

libjvm.so: cannot open shared object file: No such file or directory

at this web site I have found that (bottom part)
http://rwiki.sciviews.org/doku.php?id=packages:cran:rjava

that 



Unix: if you encounter a message similar to this one: 
Error in dyn.load(x, as.logical(local), as.logical(now)) :  unable to load 
shared library '/usr/local/lib/R/site-library/rJava/libs/rJava.so': libjvm.so: 
cannot open shared object file: No such file or directory
the R was configured without Java support. Update to R 2.4.0, make sure that 
the java command is available and  and run R CMD javareconf as root. 



I have already installed any java package in my system + now I was a bout to 
see what is my current running R version


platform   x86_64-suse-linux-gnu    
arch   x86_64   
os linux-gnu    
system x86_64, linux-gnu    
status  
major  2    
minor  15.0 
year   2012 
month  03   
day    30   
svn rev    58871    
language   R    
version.string R version 2.15.0 (2012-03-30)



so I decided to install the R2.4 as the package developer ask for...
Could you please inform me if such version even exists_

Alex
[[alternative HTML version deleted]]

__
R-help@r-project.org mailing list
https://stat.ethz.ch/mailman/listinfo/r-help
PLEASE do read the posting guide http://www.R-project.org/posting-guide.html
and provide commented, minimal, self-contained, reproducible code.


[R] hmd.mx

2012-05-17 Thread krmartina
I have trouble using function hmd.mx
I have all packages required.
I call function:

slovenia - hmd.mx(SVN, username, password, Slovenia) :

and I get this error:
  NAs introduced by coercion

And if I then call slovenia, I get:

 slovenia
Mortality data for Slovenia
Series: female male total
Years: 1983 - 2009
Ages:  0 - 110 

But I do not get any value (mortality, ...):
 slovenia$ages
NULL
 slovenia$years
NULL

I tried also for two different countries and I got the same result.

Can please anybody help me with this problem?

Thank you very much.
Martina






--
View this message in context: 
http://r.789695.n4.nabble.com/hmd-mx-tp4630350.html
Sent from the R help mailing list archive at Nabble.com.

__
R-help@r-project.org mailing list
https://stat.ethz.ch/mailman/listinfo/r-help
PLEASE do read the posting guide http://www.R-project.org/posting-guide.html
and provide commented, minimal, self-contained, reproducible code.


[R] MANOVA with random factor

2012-05-17 Thread David Costantini
Dear All
I would need to perform a MANOVA with both fixed (group, sex, group*sex) and
random (brood) effects. I wonder if this is at all possible and if R does that. 
At the moment, I only know that I can run a classic MANOVA with R.

Thank you
David

__ 

David Costantini, PhD

http://www.davidcostantini.it
NERC Postdoctoral research associate

Institute of Biodiversity, Animal Health and Comparative Medicine
School of Life Sciences
College of Medical, Veterinary and Life Sciences
University of Glasgow
Graham Kerr Building, room 511
Glasgow G12 8QQ, UK

See also my association Ornis italica
http://www.ornisitalica.com
http://www.birdcam.it





[[alternative HTML version deleted]]

__
R-help@r-project.org mailing list
https://stat.ethz.ch/mailman/listinfo/r-help
PLEASE do read the posting guide http://www.R-project.org/posting-guide.html
and provide commented, minimal, self-contained, reproducible code.


Re: [R] survival survfit with newdata

2012-05-17 Thread David Winsemius


On May 17, 2012, at 2:20 AM, Damjan Krstajic wrote:



Thanks David for prompt reply. I agree with you. However, I still  
fail to get the survfit function to work with newdata. In my  
previous example I changed the column names of testX matrix and I  
still fail.



colnames(testX)-names(coxph.model$coefficients)
sfit- survfit(coxph.model,newdata=data.frame(testX))
Error in model.frame.default(formula = Surv(trainTime, trainStatus)  
~  :

  variable lengths differ (found for 'trainX')


I don't get that error when I run this. I do get better results using  
a data argument to the coxph call. You should be getting predicted  
survival curves for 10 cases that will be estimated at the same time  
points as were available in the input data in the original data.


 coxph.model-coxph(Surv(trainTime,trainStatus)~ . ,  
data=data.frame(trainX))

 colnames(testX)-names(coxph.model$coefficients)
 sfit- survfit(coxph.model,newdata=data.frame(testX))
 plot(sfit)  # 10 curves

I do not see matrix input to coxph as a described data input, so  
perhaps you should follow the help page more closely?


--
David.



What would be solution in my simple example to get the survival  
curves for testX? Thanks in advance. DK



CC: r-help@r-project.org
From: dwinsem...@comcast.net
To: dkrsta...@hotmail.com
Subject: Re: [R] survival survfit with newdata
Date: Thu, 17 May 2012 00:52:55 -0400


On May 16, 2012, at 5:08 PM, Damjan Krstajic wrote:



Dear all,

I am confused with the behaviour of survfit with newdata option.


Yes. It has the same behavior as any other newdata/predict from
regression. You need to supply a dataframe with the same names as in
the original formula. Doesn't look as though that strategy is being
followed. The name of the column needs to be 'trainX' since that was
what was its name on the RHS of hte formula,  and you may want to
specify times. If you fail to follow those rules, the function falls
back on offering estimates from the original data.



I am using the latest version R-2-15-0. In the simple example below
I am building a coxph model on 90 patients and trying to predict 10
patients. Unfortunately the survival curve at the end is for 90
patients.


As is proper with a malformed newdata argument.


Could somebody please from the survival package confirm that this
behaviour is as expected or not - because I cannot find a way of
using 'newdata' with really new data. Thanks in advance. DK


x-matrix(rnorm(100*20),100,20)





time-runif(100,min=0,max=7)




status-sample(c(0,1), 100, replace = TRUE)

trainX-x[11:100,]


trainTime-time[11:100]



trainStatus-status[11:100]



testX-x[1:10,]

coxph.model-

coxph(Surv(trainTime,trainStatus)~ trainX)

sfit- survfit(coxph.model,newdata=data.frame(testX))





dim(sfit$surv)

[1] 90 90



[[alternative HTML version deleted]]

__
R-help@r-project.org mailing list
https://stat.ethz.ch/mailman/listinfo/r-help
PLEASE do read the posting guide http://www.R-project.org/posting-guide.html
and provide commented, minimal, self-contained, reproducible code.


David Winsemius, MD
West Hartford, CT





David Winsemius, MD
West Hartford, CT

__
R-help@r-project.org mailing list
https://stat.ethz.ch/mailman/listinfo/r-help
PLEASE do read the posting guide http://www.R-project.org/posting-guide.html
and provide commented, minimal, self-contained, reproducible code.


Re: [R] oldlogspline probabilities

2012-05-17 Thread David Winsemius


On May 17, 2012, at 3:16 AM, Beutel, Terry S wrote:

I using oldlogspline (from logspline package) to model data  
distributions, and having a problem.


My data are search area sizes. They are based on circular search  
radii from random points to the nearest edge of the nearest grass  
tussock. Search area sizes are distributed from 0 (the random point  
intercepts a tussock) and upwards (as points are further from any  
tussocks). The density of all my distributions (using doldlogspline)  
are highest at size=0, and decline as search area increases.


I am most interested in the probability of a value in the  
distribution being equal to 0 (ie probability of a direct hit on a  
tussock). I know I can just use the proportion of actual hits, but  
am curious to compare this to an estimate from a density estimation.  
Unfortunately when using poldlogspline, this probability is always=0  
(simulated data example is below). How can this be, given that the  
density is highest at area=0?



simdat-c(rep(0,8),rexp(92))
myspline-oldlogspline(simdat,lbound=0)
poldlogspline(fit=myspline, q=0)

[1] 0

Any help to work out the probability of an area value in my  
distribution = 0 would be appreciated


Probabilities often are zero over a zero-length interval , even if  
their densities are positive. Surely this tussock has a finite width?


 logspline::poldlogspline(fit=myspline, q=0.01)
[1] 0.04138533

--
David Winsemius, MD
West Hartford, CT

__
R-help@r-project.org mailing list
https://stat.ethz.ch/mailman/listinfo/r-help
PLEASE do read the posting guide http://www.R-project.org/posting-guide.html
and provide commented, minimal, self-contained, reproducible code.


Re: [R] Error on easy way for JoSAE Package

2012-05-17 Thread David Winsemius


On May 17, 2012, at 2:26 AM, ana24maria wrote:


I have attached to this message the first 20 lines of the output for
dput(amigo).


If ypu attempted to send something,  the mail-server scrubbed it.


Unfortunately, i can't send it the overall output.
Thank you.


I did not use the word attach.

 When you get around to reading the Posting Guide, pay particular  
attention to the section Technical details of posting. Sometimes  
person posting from Nabble will have success putting things up on that  
website.


--
David.



On Wed, May 16, 2012 at 6:10 PM, David Winsemius [via R] 
ml-node+s789695n4630253...@n4.nabble.com wrote:



On May 16, 2012, at 1:33 AM, ana24maria wrote:


Thank you very much.
After using dput and the easy way ( result -
eblup.mse.f.wrap(domain.data
= amigo, lme.obj = fit.lme)),
i have got the following error:

Error in `[.data.frame`(sample.data, , variabs) :
undefined columns selected


What John was asking you to do was at your console just type:

dput(amigo)

... and then copy the output to an email and send that to the list.
Your first posting had data that was ambiguous as to content as well
as mangled by the various email clients and servers that processed on
the path to our eyes.



What should I do?


You should also read the Posting Guide.





David Winsemius, MD
West Hartford, CT

__
R-help@r-project.org mailing list
https://stat.ethz.ch/mailman/listinfo/r-help
PLEASE do read the posting guide http://www.R-project.org/posting-guide.html
and provide commented, minimal, self-contained, reproducible code.


Re: [R] Complex sort problem

2012-05-17 Thread David Winsemius


On May 17, 2012, at 6:45 AM, Axel Urbiz wrote:


Dear List,

Is there a way I can sort a sample based on a sort index constructed  
from
the data from which the sample is taken? Basically, I need to take  
'many'

samples from the same source data and sort them. This can be very time
consuming for long vectors. Is there any way I can sort the data  
only once

initially, and use that sort order for the samples?

I believe that idea is what is implemented in tree-based  
classifiers, so
the data is sorted only once initially and that sort order is used  
for the

child nodes.


set.seed(12345)
x - sample(0:100, 10)
x.order - order(x)
x.sorted - x[x.order]

sample.ind - sample(1:length(x), 5, replace = TRUE)  #sample 1/2  
size with

replacement
x.sample - x[sample.ind]

x.sample.sorted -   #??? (without sorting again)

Thanks for any help on this.



If you had created a named vector or used a dataframe with rownames  
you would have a record of the original sort order.



Regards,
Axel.

[[alternative HTML version deleted]]

__
R-help@r-project.org mailing list
https://stat.ethz.ch/mailman/listinfo/r-help
PLEASE do read the posting guide http://www.R-project.org/posting-guide.html
and provide commented, minimal, self-contained, reproducible code.


David Winsemius, MD
West Hartford, CT

__
R-help@r-project.org mailing list
https://stat.ethz.ch/mailman/listinfo/r-help
PLEASE do read the posting guide http://www.R-project.org/posting-guide.html
and provide commented, minimal, self-contained, reproducible code.


Re: [R] Add column from other columns data.

2012-05-17 Thread Petr PIKAL
 Something along the lines of 
 
 dat2  -  ifelse( dat1==1 , yes, no)

Another option is in this case

dat2  -  c(no, yes)[dat1+1]

Regards
Petr

 
 should do it.
 
 John Kane
 Kingston ON Canada
 
 
  -Original Message-
  From: s1010...@student.hsleiden.nl
  Sent: Mon, 14 May 2012 05:45:38 -0700 (PDT)
  To: r-help@r-project.org
  Subject: [R] Add column from other columns data.
  
  Hi everyone,
  
  I am having some problems with making a new colomn wit data in it.
  I have this one column named: Fulfilled
  
  Fulfilled
  1
  1
  0
  1
  1
  1
  1
  0
  0
  1
  
  And now I would like to add another colum to my .csv file (Finished)
  
  In this Finished column I would like to have Yes or No.
  Where in colomn Fullfilled is a 1, Finished should have a Yes.
  Like this:
  
  Fullfilled Finished
  1 Yes
  1 Yes
  0 No
  etc
  
  Now I know how to grab the data out of a column, and also know how to
  save
  data inside a .csv file.
  That is no problem.
  But how do I get the right Yes or No on the right place in the other
  column?
  
  # Get al values: 1
  Fullfilled_1 = Fullfilled[Fullfilled = 1]
  
  I was thinkng about subset.
  But I don' t realy know if that would be realy it
  
  Maybe somebody here can push me a little in the right direction?
  
  
  --
  View this message in context:
  
http://r.789695.n4.nabble.com/Add-column-from-other-columns-data-tp4629921.html

  Sent from the R help mailing list archive at Nabble.com.
  
  __
  R-help@r-project.org mailing list
  https://stat.ethz.ch/mailman/listinfo/r-help
  PLEASE do read the posting guide
  http://www.R-project.org/posting-guide.html
  and provide commented, minimal, self-contained, reproducible code.
 
 
 Publish your photos in seconds for FREE
 TRY IM TOOLPACK at http://www.imtoolpack.com/default.aspx?rc=if4
 
 __
 R-help@r-project.org mailing list
 https://stat.ethz.ch/mailman/listinfo/r-help
 PLEASE do read the posting guide 
http://www.R-project.org/posting-guide.html
 and provide commented, minimal, self-contained, reproducible code.

__
R-help@r-project.org mailing list
https://stat.ethz.ch/mailman/listinfo/r-help
PLEASE do read the posting guide http://www.R-project.org/posting-guide.html
and provide commented, minimal, self-contained, reproducible code.


Re: [R] hu6800cdf

2012-05-17 Thread Martin Morgan

Hi Srod --

On 05/16/2012 11:04 PM, srod wrote:

Hi,

I'm using a command in bioconductor that seems to require a package called
hu6800cdf. I've installed this properly but I still get the same error:

  Could not find array definition file ' hu6800cdf.qcdef '. Simpleaffy does
not know the QC parameters for this array type.
See the package vignette for details about how to specify QC parameters
manually.


Please ask questions about Bioconductor packages on the Bioconductor 
mailing list,


  http://bioconductor.org/help/mailing-list/

cc'ing the maintainer

   packageDescription(simpleaffy)$Maintainer

including the output of

   sessionInfo()

and the precise command that generates the error.

The package vignette

   vignette(package=simpleaffy, simpleAffy)

especially section 5.4, Specifying alternate QC configurations, may be 
helpful.


Martin



I've tried specifying the cdfname file by using

data-ReadAffy(cdfname=hu6800cdf)  but it still returns the same error.

The version of R that I'm using is 2.15 and the bioconductor version is
2.10.

Does anyone know how to solve this?

Thanks,

srod

--
View this message in context: 
http://r.789695.n4.nabble.com/hu6800cdf-tp4630337.html
Sent from the R help mailing list archive at Nabble.com.

__
R-help@r-project.org mailing list
https://stat.ethz.ch/mailman/listinfo/r-help
PLEASE do read the posting guide http://www.R-project.org/posting-guide.html
and provide commented, minimal, self-contained, reproducible code.



--
Computational Biology
Fred Hutchinson Cancer Research Center
1100 Fairview Ave. N. PO Box 19024 Seattle, WA 98109

Location: M1-B861
Telephone: 206 667-2793

__
R-help@r-project.org mailing list
https://stat.ethz.ch/mailman/listinfo/r-help
PLEASE do read the posting guide http://www.R-project.org/posting-guide.html
and provide commented, minimal, self-contained, reproducible code.


Re: [R] Is there R 2.4 version_

2012-05-17 Thread Sarah Goslee
Hi Alaios,

The web page you're quoting was written in 2006. There *was* an R 2.4,
but it's no longer cutting edge, to say the least.

Earlier in the document it says R 2.4 or higher, which is what you've
got. I'd first look for newer documentation, then try the instructions
given with the current version of R you already have installed.

Sarah

On Thu, May 17, 2012 at 7:29 AM, Alaios ala...@yahoo.com wrote:
 Dear all,
 I am trying to install the rJava package.
 I am getting the following message

 libjvm.so: cannot open shared object file: No such file or directory

 at this web site I have found that (bottom part)
 http://rwiki.sciviews.org/doku.php?id=packages:cran:rjava

 that



 Unix: if you encounter a message similar to this one:
 Error in dyn.load(x, as.logical(local), as.logical(now)) :  unable to load 
 shared library '/usr/local/lib/R/site-library/rJava/libs/rJava.so': 
 libjvm.so: cannot open shared object file: No such file or directory
 the R was configured without Java support. Update to R 2.4.0, make sure that 
 the java command is available and  and run R CMD javareconf as root.



 I have already installed any java package in my system + now I was a bout to 
 see what is my current running R version


 platform   x86_64-suse-linux-gnu
 arch   x86_64
 os linux-gnu
 system x86_64, linux-gnu
 status
 major  2
 minor  15.0
 year   2012
 month  03
 day    30
 svn rev    58871
 language   R
 version.string R version 2.15.0 (2012-03-30)



 so I decided to install the R2.4 as the package developer ask for...
 Could you please inform me if such version even exists_

 Alex

-- 
Sarah Goslee
http://www.functionaldiversity.org

__
R-help@r-project.org mailing list
https://stat.ethz.ch/mailman/listinfo/r-help
PLEASE do read the posting guide http://www.R-project.org/posting-guide.html
and provide commented, minimal, self-contained, reproducible code.


Re: [R] Unable to install package

2012-05-17 Thread Uwe Ligges



On 17.05.2012 00:49, Rismyname wrote:

Hi,
I get the following error while installing a package. Can someone please
help?

install.packages(memisc)
Warning in install.packages :
   argument 'lib' is missing: using 'C:/Users/ravi/Documents/R/R-2.15.0'
Warning in install.packages :
   downloaded length 8255 != reported length 200
Error in install.packages : Line starting '!DOCTYPE html PUBLI ...' is
malformed!



OS? Version of R? Selected CRAN mirror?

Anyway: R tried to download the package but got an html page, obviously, 
hence either the mirror you are using is corrupted or somone in between 
(like some proxy?) delivers html pages rather than packages...


Uwe Ligges







thanks

--
View this message in context: 
http://r.789695.n4.nabble.com/Unable-to-install-package-tp4630320.html
Sent from the R help mailing list archive at Nabble.com.

__
R-help@r-project.org mailing list
https://stat.ethz.ch/mailman/listinfo/r-help
PLEASE do read the posting guide http://www.R-project.org/posting-guide.html
and provide commented, minimal, self-contained, reproducible code.


__
R-help@r-project.org mailing list
https://stat.ethz.ch/mailman/listinfo/r-help
PLEASE do read the posting guide http://www.R-project.org/posting-guide.html
and provide commented, minimal, self-contained, reproducible code.


Re: [R] (bug?) in survfit in R-2-15.0

2012-05-17 Thread Terry Therneau
The predict methods are designed to work with a dataframe.  In the case 
of survfit(cox model) the code has, for backwards compatability 
purposes, the ability to use a vector as the newdata argument.  This 
feature is not documented in the help file, on purpose, and is expected 
to go away one day.  This backwards compatatibilty works in all my 
test cases, but not in the one you present.  This may accelerate my 
plans for removal.


  The best solution for you is to use a dataframe for both the coxph 
fit and the predict call.  Avoid using objects that are in the global 
data or are attached.  Below I reprise your example in safer code.  The 
coxph/survfit calls both work from a temporary data frame called 
dummy.  Your use of the coxph/survfit pair to get a survival curve for 
glmnet is clever, by the way.  I like it.


library(glmnet)
library(survival)
load(system.file(doc,VignetteExample.rdata,package=glmnet))

fit - with(patient.data, glmnet(x[-1,], Surv(time[-1], status[-1]),
 family=cox, alpha=.05, maxit=1))
max.dev.index - which.max(fit$dev.ratio)
optimal.beta - fit$beta[, max.dev.index]
nonzero.coef - (optimal.beta != 0)

dummy - with(patient.data, data.frame(time=time, status=status, 
x=x[,nonzero.coef]))

coxfit - coxph(Surv(time, status) ~ ., data=dummy, subset= -1,
iter=0, init=optimal.beta[nonzero.coef])
sfit - survfit(coxfit, newdata=dummy[1,])

Terry Therneau

 begin included message ---

Dear all,

I am using glmnet + survival and due to the latest
release of glmnet 1.7.4 I was forced to use the latest version of R 2.15.0.
My previous version of R was 2.10.1. I changed glmnet version and R
version and when I started to get weird results I was not sure where the 
bug was.


After putting glmnet back to previous version, I have
found that the bug or weird behaviour happens in survival survfit. My 
script is

below in email and it is uses glmnet 1.7.3. I get two completely different
answers in different versions of R and in my opinion the older version of
survival package returns correct value.

in R 2-10-1 I get

?
cat(dim(sfit$surv))

?

?
cat(length(sfit$surv))

32

?

?

in R-2-15-0 I get

cat(dim(sfit$surv))

62 99

?
cat(length(sfit$surv))

6138

?

?Kind regardsDK


library(survival)

library(glmnet)

load(system.file(doc,VignetteExample.rdata,package=glmnet))

attach(patient.data)

trainX?
-x[-1,]

trainTime??
-time[-1]

trainStatus - status[-1]

fit -

glmnet(trainX,Surv(trainTime,trainStatus),family=cox,alpha=0.5,maxit=1)

max.dev.index
- which.max(fit$dev.ratio)

optimal.lambda - fit$lambda[max.dev.index]
optimal.beta? -
fit$beta[,max.dev.index] nonzero.coef - abs(optimal.beta)0 selectedBeta
- optimal.beta[nonzero.coef]

selectedTrainX??
- trainX[,nonzero.coef]

coxph.model- coxph(Surv(trainTime,trainStatus)~

selectedTrainX,init=selectedBeta,iter=0)

selectedTestX - x[1,nonzero.coef]

sfit- survfit(coxph.model,newdata=selectedTestX)

cat(\ndim(sfit$surv))

cat(dim(sfit$surv))

cat(\nlength(sfit$surv))

cat(length(sfit$surv))

__
R-help@r-project.org mailing list
https://stat.ethz.ch/mailman/listinfo/r-help
PLEASE do read the posting guide http://www.R-project.org/posting-guide.html
and provide commented, minimal, self-contained, reproducible code.


Re: [R] max value

2012-05-17 Thread Petr PIKAL
Hi

 
 On 2012-05-15 08:36, Melissa Rosenkranz wrote:
  Here is an R problem I am struggling with:
  My dataset is organized like this...
 
  subject   sessionvariable_x   variable_y
  01 11interger values
  01 12
  01 13
  01 21
  01 22
  01 23
  02 11
  02 12
  02 13
  02 21
  02 22
  02 23
  03 11
  03 12
  03 13
  03 21
  03 22
  03 23
  ...
 
  I need to find the level of variable x at which variable y has the 
maximum
  value for each individual for each session. Then, I need to create 
another
  variable, say variable z that labels that row in the dataset as the 
max
  for that individual at that time. I have searched the archives and the 
web
  for ideas, but am having trouble finding appropriate search terms for 
what
  I need to do. Any advice? Thank you!!
 
 
 This is one way:

Here is another

d$z-ave(d$y, d$subject, d$session, FUN=function(x) x==max(x))

Regards
Petr

 
set.seed(123)
d - data.frame(
 subject = gl(3,6,labels=c(01,02,03)),
 session = gl(2,3,18),
 x = gl(3,1,18),
 y = sample(11:15, 18, replace=TRUE))
 
library(plyr)
ddply(d, .(subject, session), transform,
  z = ifelse(y == max(y), 1, 0))
 
 Peter Ehlers
 
 __
 R-help@r-project.org mailing list
 https://stat.ethz.ch/mailman/listinfo/r-help
 PLEASE do read the posting guide 
http://www.R-project.org/posting-guide.html
 and provide commented, minimal, self-contained, reproducible code.

__
R-help@r-project.org mailing list
https://stat.ethz.ch/mailman/listinfo/r-help
PLEASE do read the posting guide http://www.R-project.org/posting-guide.html
and provide commented, minimal, self-contained, reproducible code.


Re: [R] replacing with NA

2012-05-17 Thread Petr PIKAL
 
 x[is.na(z)] - NA
 
 This might send you a nasty bug if x and z are different lengths
 though -- just a head's up.

Another option
x*!is.na(z)*z

Regards
Petr


 
 Michael
 
 On Wed, May 16, 2012 at 12:55 PM, Mintewab Bezabih
 mintewab.beza...@economics.gu.se wrote:
  Dear R users,
 
  I was wondering  how I can replace the values of a vector with the 
 values from in another vector in the same row
 
  For example, how can I replace the value of x below with NA when the 
 value of Z in the same row is NA?
  x -1:20
  z- c(11, 15, 17, 2, 18, 6, 7, NA, 12, 10,21, 25, 27, 12, 28, 16,17, 
NA, 12, 10)
 
 
  Many thanks
  Mintewab
 
  
  Från: Mintewab Bezabih
  Skickat: den 15 maj 2012 15:53
  Till: r-help@r-project.org
  Kopia: r-help@r-project.org
  Ämne: missing observations
 
  Dear R users,
 
  I have missing observations in my data that I remove in my analysis. I 

 am able to run my codes alright but I want the non missing values to be 
 correctly identified and therefore want to tag my id vector along in my 
 results. Since the vector of ids has no role in the analysis, I dont 
know 
 how to include it.
 
 
 
  Here is my reprducable example:and my id is the vector I want to add 
to 
 the analysis somehow so that my missing values are identified. I cannot 
 use  na.action function and that is why I have to drop my missing 
 obesevations beforehand.
 
 
  library(fields)
  x -1:20
  y- runif(20)
  z- c(11, 15, 17, 2, 18, 6, 7, NA, 12, 10,21, 25, 27, 12, 28, 16,
  17, NA, 12, 10)
  id -1:20
 
  mydataset-data.frame(x, y, z)
  temperature[complete.cases(mydataset),]
 
   x- temperature[, c(1)]
  y- temperature[, c(2)]
  z- temperature[, c(3)]
 
  tpsfit - Tps(cbind(x, y), z, scale.type=unscaled)
 
 
 
 
  Many thanks as always.
  Regards,
  Mintewab
 
  __
  R-help@r-project.org mailing list
  https://stat.ethz.ch/mailman/listinfo/r-help
  PLEASE do read the posting guide 
http://www.R-project.org/posting-guide.html
  and provide commented, minimal, self-contained, reproducible code.
 
 __
 R-help@r-project.org mailing list
 https://stat.ethz.ch/mailman/listinfo/r-help
 PLEASE do read the posting guide 
http://www.R-project.org/posting-guide.html
 and provide commented, minimal, self-contained, reproducible code.

__
R-help@r-project.org mailing list
https://stat.ethz.ch/mailman/listinfo/r-help
PLEASE do read the posting guide http://www.R-project.org/posting-guide.html
and provide commented, minimal, self-contained, reproducible code.


[R] ctree for suvival analysis problem

2012-05-17 Thread ofraam
Hi All,

I'm using the party package to grow conditional inference trees for survival
analysis. 
When I used party version  party_0.9-9991 everything worked well, but when I
update to party_1.0-2 (due to using 64bit R), I get an error. For simplicity
I will show the error I get for the example in the party documentation:
### survival analysis
if (require(ipred)) {
data(GBSG2, package = ipred)
GBSG2ct - ctree(Surv(time, cens) ~ .,data = GBSG2)
plot(GBSG2ct)
treeresponse(GBSG2ct, newdata = GBSG2[1:2,])
}

for the plot(GBSG2ct) line I get the following error:
Error in Summary.Surv(c(1814, 2018, 712, 1807, 772, 448, 2172, 2161, 471,  : 
  Invalid operation on a survival time

and for treeresponse(GBSG2ct, newdata = GBSG2[1:2,]) I get this error:
Error: extends(class(y), Surv) is not TRUE

(I care more about treeresponse but assume it is related).

Any help would be highly appreciated!

Thanks,
Ofra

--
View this message in context: 
http://r.789695.n4.nabble.com/ctree-for-suvival-analysis-problem-tp4630362.html
Sent from the R help mailing list archive at Nabble.com.

__
R-help@r-project.org mailing list
https://stat.ethz.ch/mailman/listinfo/r-help
PLEASE do read the posting guide http://www.R-project.org/posting-guide.html
and provide commented, minimal, self-contained, reproducible code.


[R] [R-pkgs] new version of fda fixes bug in pca.fd and supports library(Matrix)

2012-05-17 Thread Spencer Graves

Hello, All:


	  fda_2.2.8 (functional data analysis) is now available on CRAN.  This 
revision includes the following improvements:



1.  A bug in pca.fd has been fixed.


		2.  Many functions have a new argument returnMatrix, which if 
TRUE allows the function to use the sparse matrix representations in 
library(Matrix).  This should allow users to solve some larger problems 
and provides a speed advantage in a few cases.



		3.  A few other minor bugs have been fixed including problems with 
some of the script files in system.files('scripts', package='fda').



	  A package using the returnMatrix argument will also need Matrix in 
the dependencies in DESCRIPTION and something like import(Matrix) in 
NAMESPACE.  Each use will require adding returnMatrix = TRUE to an 
appropriate fda function call.



  Best Wishes,
Jim Ramsay, Giles Hooker, Spencer Graves

___
R-packages mailing list
r-packa...@r-project.org
https://stat.ethz.ch/mailman/listinfo/r-packages

__
R-help@r-project.org mailing list
https://stat.ethz.ch/mailman/listinfo/r-help
PLEASE do read the posting guide http://www.R-project.org/posting-guide.html
and provide commented, minimal, self-contained, reproducible code.


[R] Importing ASCII flat

2012-05-17 Thread Richard Iles
I am trying to import Indian National Sample Survey Data. It is ASCII flat
and an example is below

  00146030602500111710107111201*01*00211 270104070204093
00246030602500111710107111201*02*00806104910519572 022  2600
11503055   170  4005  00346030602500111710107111201*03*
111039204112  222

Previous post don't seem to provide much help. I have an document outlining
the layout of the data, but don't know how to start. An example of the
layout is that the values in bold indicate separate levels.Example of
layout is below:

  Sl.No. Item Blk Item Col Len Byte Position Remarks  1 Common Items
  33 1  - 33 Auto-duplicated  2 Level  2 34  - 35  02 Generated  3 Filler
5 36  - 40 0 Generated  4 HHS Size 3 1 2 41  - 42  5 NIC Code(5-digit)
3 2 5 43  - 47  6 NCO Code(3-digit) 3 3 3 48  - 50  7 HHS type 3 4 1 51  -
51  8 Religion 3 5 1 52  - 52
As is obvious, I am relatively new to R. Any constructive advice would be
welcome.

Richard Iles
-- 
Economics Ph.D. student
Delhi School of Economics (visiting)
e: richard.i...@griffithuni.edu.au

website: 
https://sites.google.com/a/griffithuni.edu.au/richard-iles/homehttps://sites.google.com/site/richardileshome/home

[[alternative HTML version deleted]]

__
R-help@r-project.org mailing list
https://stat.ethz.ch/mailman/listinfo/r-help
PLEASE do read the posting guide http://www.R-project.org/posting-guide.html
and provide commented, minimal, self-contained, reproducible code.


Re: [R] error code trying to extract second column from coeftest output

2012-05-17 Thread Petr PIKAL
Hi

 
 I want to use the standard error values in the summary that is produced 
using
 coeftest, but I am getting an error code- any ideas?

See what is structure of coeftest object by

str(coeftest(lmodT_WBHO))

and from this you shall deduct how to select second column.

Regards
Petr


 
  library(lmtest)
  coeftest(lmodT_WBHO)
 
 t test of coefficients:
 
 Estimate Std. Error t value  Pr(|t|) 
 t1W  5.948190.17072 34.8410  2.2e-16 ***
 t2W  6.562160.17438 37.6322  2.2e-16 ***
 t3W  6.082520.16525 36.8082  2.2e-16 ***
 t4W  6.180410.17028 36.2949  2.2e-16 ***
 t1B  5.50.50566 10.8768  2.2e-16 ***
 t2B  5.650000.53034 10.6535  2.2e-16 ***
 t3B  4.523810.51756  8.7406  2.2e-16 ***
 t4B  4.380950.51756  8.4646  2.2e-16 ***
 t1H  5.050000.53034  9.5221  2.2e-16 ***
 t2H  4.80.55903  8.5465  2.2e-16 ***
 t3H  5.526320.54412 10.1564  2.2e-16 ***
 t4H  4.714290.63388  7.4372 2.236e-13 ***
 t1O  5.176470.57524  8.9988  2.2e-16 ***
 t2O  5.818180.50566 11.5060  2.2e-16 ***
 t3O  6.50.63388 10.2543  2.2e-16 ***
 t4O  5.714290.63388  9.0147  2.2e-16 ***
 ---
 Signif. codes:  0 ‘***’ 0.001 ‘**’ 0.01 ‘*’ 0.05 ‘.’ 0.1 ‘ ’ 1 
 
  se1 - coeftest(lmodT_WBHO)$coef[,2]
 Error in coeftest(lmodT_WBHO)$coef : 
   $ operator is invalid for atomic vectors
  
 
 
 --
 View this message in context: http://r.789695.n4.nabble.com/error-code-
 trying-to-extract-second-column-from-coeftest-output-tp4630298.html
 Sent from the R help mailing list archive at Nabble.com.
 
 __
 R-help@r-project.org mailing list
 https://stat.ethz.ch/mailman/listinfo/r-help
 PLEASE do read the posting guide 
http://www.R-project.org/posting-guide.html
 and provide commented, minimal, self-contained, reproducible code.

__
R-help@r-project.org mailing list
https://stat.ethz.ch/mailman/listinfo/r-help
PLEASE do read the posting guide http://www.R-project.org/posting-guide.html
and provide commented, minimal, self-contained, reproducible code.


[R] how to download source code, pdf manual for any package from internet?

2012-05-17 Thread sagarnikam123
i can get list of available packages by
 r-available.packages()

No of available packages
 length(r[,1])
[1] 3739

single package name
 r[1,1]
[1] ACCLMA

i want download all packages  starting with A / a alphabets,
with their source code, pdf manual/vignettes from internet


--
View this message in context: 
http://r.789695.n4.nabble.com/how-to-download-source-code-pdf-manual-for-any-package-from-internet-tp4630354.html
Sent from the R help mailing list archive at Nabble.com.

__
R-help@r-project.org mailing list
https://stat.ethz.ch/mailman/listinfo/r-help
PLEASE do read the posting guide http://www.R-project.org/posting-guide.html
and provide commented, minimal, self-contained, reproducible code.


Re: [R] as.function parameters

2012-05-17 Thread jackl
Hi..

Ok here is an example on how I wanted the tree to be implemented in R:

-  the tree is, as you wrote, saved as a list of different tree levels
- each tree level is also saved as a list of different nodes in that
specific level
- and for the last part, each node is then saved as a list of functions

example:
tree - list(root, lvl1, lvl2)
root - list(node00)
lvl1 - list(node10, node11)
lvl2 - list(node20, node21, node22)

node00 - list(f1,f2,f3)
node10 - list(f1,f2,f3)
node11 - list(f1,f2,f3)
..

note: I wrote f1, f2 and f3 in each node because it is the same function,
just with the different parameter, the stock price at that node.

I tried implementing a tree manually and I found out that the
independences between one node and each childnode cause a
heavy computation power.. (the function f3 contains f3 of the
two childnodes and so on..)
example:
node11$f3 - max(node11$f2, node21$f3, node22$f3)

Im facing this problem even with a tree with 'only' 4-5 layers.. 

best thanks for any answers

--
View this message in context: 
http://r.789695.n4.nabble.com/as-function-parameters-tp4620390p4630353.html
Sent from the R help mailing list archive at Nabble.com.

__
R-help@r-project.org mailing list
https://stat.ethz.ch/mailman/listinfo/r-help
PLEASE do read the posting guide http://www.R-project.org/posting-guide.html
and provide commented, minimal, self-contained, reproducible code.


Re: [R] how to find outliers from the list of values

2012-05-17 Thread Petr PIKAL
Hi

I had not see any answer yet but maybe there is nobody who wants to touch 
the elusive object of outlier. Neither me, but here are some ideas how 
one can proceed.

First of all its always up to you what is considered an outlier and how 
will you deal with them. 

I usually call an outlier any item which does not fit to the pattern and 
the pattern is usually best observed by some plotting function. You can 
identify outlier points, inspect the data source, correct typing mistakes 
and only if the value is really measured and you can not find any reason 
why it has such value it is real outlier. Then ***you*** need to decide 
what to do with it - discard, can come from some long tailed distribution, 
...

So here are my 0.02$ regarding an outlier theme.

Regards
Petr

 
 Hi,
 I am new to R and I would like to get your help in finding
 'outliers'.
  I have mvoutlier package installed in my system and added the package .
 But I not able find a function from 'mvoutlier' package which will 
identify
 'outliers'.
 This is the sample list of data I have got which has one out-lier.
11489  11008  11873  8000  9558  8645  8024  8371  It will be of
 great help if somebody have got an example script for the same.
 
 Thanks  Regards,
 Thomas
 
[[alternative HTML version deleted]]
 
 __
 R-help@r-project.org mailing list
 https://stat.ethz.ch/mailman/listinfo/r-help
 PLEASE do read the posting guide 
http://www.R-project.org/posting-guide.html
 and provide commented, minimal, self-contained, reproducible code.

__
R-help@r-project.org mailing list
https://stat.ethz.ch/mailman/listinfo/r-help
PLEASE do read the posting guide http://www.R-project.org/posting-guide.html
and provide commented, minimal, self-contained, reproducible code.


[R] neighbour list in lm.LMtests

2012-05-17 Thread johnnyzn
Dear everyone! I am a new user in R. Recently, I have been struggling to work
out lagrange multiplier test in R, after read the description in R. I am
still confusing with method of assigning a neighbour list to listw.
Many thanks if anyone can give me a hint 

--
View this message in context: 
http://r.789695.n4.nabble.com/neighbour-list-in-lm-LMtests-tp4630366.html
Sent from the R help mailing list archive at Nabble.com.

__
R-help@r-project.org mailing list
https://stat.ethz.ch/mailman/listinfo/r-help
PLEASE do read the posting guide http://www.R-project.org/posting-guide.html
and provide commented, minimal, self-contained, reproducible code.


Re: [R] ctree for suvival analysis problem

2012-05-17 Thread Achim Zeileis

On Thu, 17 May 2012, ofraam wrote:


Hi All,

I'm using the party package to grow conditional inference trees for survival
analysis.
When I used party version  party_0.9-9991 everything worked well, but when I
update to party_1.0-2 (due to using 64bit R), I get an error. For simplicity
I will show the error I get for the example in the party documentation:
### survival analysis
if (require(ipred)) {
data(GBSG2, package = ipred)
GBSG2ct - ctree(Surv(time, cens) ~ .,data = GBSG2)
plot(GBSG2ct)
treeresponse(GBSG2ct, newdata = GBSG2[1:2,])
}


Works for me without a problem.


for the plot(GBSG2ct) line I get the following error:
Error in Summary.Surv(c(1814, 2018, 712, 1807, 772, 448, 2172, 2161, 471,  :
 Invalid operation on a survival time

and for treeresponse(GBSG2ct, newdata = GBSG2[1:2,]) I get this error:
Error: extends(class(y), Surv) is not TRUE

(I care more about treeresponse but assume it is related).


My guess is that you have loaded package rms but haven't told us so. 
This overwrites Surv() in a way that is incompatible with the usage in 
ctree().


Best,
Z



Any help would be highly appreciated!

Thanks,
Ofra

--
View this message in context: 
http://r.789695.n4.nabble.com/ctree-for-suvival-analysis-problem-tp4630362.html
Sent from the R help mailing list archive at Nabble.com.

__
R-help@r-project.org mailing list
https://stat.ethz.ch/mailman/listinfo/r-help
PLEASE do read the posting guide http://www.R-project.org/posting-guide.html
and provide commented, minimal, self-contained, reproducible code.



__
R-help@r-project.org mailing list
https://stat.ethz.ch/mailman/listinfo/r-help
PLEASE do read the posting guide http://www.R-project.org/posting-guide.html
and provide commented, minimal, self-contained, reproducible code.


Re: [R] Importing ASCII flat

2012-05-17 Thread Uwe Ligges



On 17.05.2012 14:17, Richard Iles wrote:

I am trying to import Indian National Sample Survey Data. It is ASCII flat
and an example is below

   00146030602500111710107111201*01*00211 270104070204093
00246030602500111710107111201*02*00806104910519572 022  2600
11503055   170  4005  00346030602500111710107111201*03*
111039204112  222



Do you have a file format specification? This is probably some fixed 
width formatted file, hence read.fwf may help to import the data. See 
?read.fwf and the R Data Import/Export manual.


Best,
Uwe Ligges



Previous post don't seem to provide much help. I have an document outlining
the layout of the data, but don't know how to start. An example of the
layout is that the values in bold indicate separate levels.Example of
layout is below:

   Sl.No. Item Blk Item Col Len Byte Position Remarks  1 Common Items
   33 1  - 33 Auto-duplicated  2 Level  2 34  - 35  02 Generated  3 Filler
5 36  - 40 0 Generated  4 HHS Size 3 1 2 41  - 42  5 NIC Code(5-digit)
3 2 5 43  - 47  6 NCO Code(3-digit) 3 3 3 48  - 50  7 HHS type 3 4 1 51  -
51  8 Religion 3 5 1 52  - 52
As is obvious, I am relatively new to R. Any constructive advice would be
welcome.

Richard Iles


__
R-help@r-project.org mailing list
https://stat.ethz.ch/mailman/listinfo/r-help
PLEASE do read the posting guide http://www.R-project.org/posting-guide.html
and provide commented, minimal, self-contained, reproducible code.


Re: [R] Is there R 2.4 version_

2012-05-17 Thread Alaios
Thanks 

Solution found


Fixed:
a. run as root the sudo R CMD javareconf
(not as simple user)

b. then install rJava from tar.gz





 From: Sarah Goslee sarah.gos...@gmail.com

Cc: R help R-help@r-project.org 
Sent: Thursday, May 17, 2012 2:44 PM
Subject: Re: [R] Is there R 2.4 version_

Hi Alaios,

The web page you're quoting was written in 2006. There *was* an R 2.4,
but it's no longer cutting edge, to say the least.

Earlier in the document it says R 2.4 or higher, which is what you've
got. I'd first look for newer documentation, then try the instructions
given with the current version of R you already have installed.

Sarah


 Dear all,
 I am trying to install the rJava package.
 I am getting the following message

 libjvm.so: cannot open shared object file: No such file or directory

 at this web site I have found that (bottom part)
 http://rwiki.sciviews.org/doku.php?id=packages:cran:rjava

 that



 Unix: if you encounter a message similar to this one:
 Error in dyn.load(x, as.logical(local), as.logical(now)) :  unable to load 
 shared library '/usr/local/lib/R/site-library/rJava/libs/rJava.so': 
 libjvm.so: cannot open shared object file: No such file or directory
 the R was configured without Java support. Update to R 2.4.0, make sure that 
 the java command is available and  and run R CMD javareconf as root.



 I have already installed any java package in my system + now I was a bout to 
 see what is my current running R version


 platform   x86_64-suse-linux-gnu
 arch   x86_64
 os linux-gnu
 system x86_64, linux-gnu
 status
 major  2
 minor  15.0
 year   2012
 month  03
 day    30
 svn rev    58871
 language   R
 version.string R version 2.15.0 (2012-03-30)



 so I decided to install the R2.4 as the package developer ask for...
 Could you please inform me if such version even exists_

 Alex

-- 
Sarah Goslee
http://www.functionaldiversity.org
[[alternative HTML version deleted]]

__
R-help@r-project.org mailing list
https://stat.ethz.ch/mailman/listinfo/r-help
PLEASE do read the posting guide http://www.R-project.org/posting-guide.html
and provide commented, minimal, self-contained, reproducible code.


Re: [R] Help needed for efficient way to loop through rows and columns

2012-05-17 Thread David L Carlson
This will accomplish what you want and should be relatively easy to modify.

# Create data.fram
names - c(S1, S2, S3, S4)
X - c(BB, AA, AB, AA)
Y - c(BB, BB, AB, AA)
Z - c(BB, BB, AA, NA)
AorB - c(A, A, B, B)
sample - data.frame(names, X, Y, Z, AorB, 
  stringsAsFactors=FALSE)

# Create recoded data.frame
samplemod - sample
samplemod[,2:4] - NA

# Recoded values into samplemod
for (i in 1:nrow(sample)) {
  for (j in 2:4) {
if (!is.na(sample[i,j])){
  if (sample[i, 5] == A) {
samplemod[i,j] - switch(sample[i,j], AA = 2, 
  AB = 1, BA = 1, BB = 0)
  }
  else { 
if (sample[i, 5] == B) {
samplemod[i,j] - switch(sample[i,j], AA = 0, 
  AB = 1, BA = 1, BB = 2)
}
  }
}
  }
}



--
David L Carlson
Associate Professor of Anthropology
Texas AM University
College Station, TX 77843-4352



From: Priya Bhatt [mailto:bhatt...@gmail.com] 
Sent: Wednesday, May 16, 2012 1:55 PM
To: dcarl...@tamu.edu; r-help@r-project.org
Subject: Re: [R] Help needed for efficient way to loop through rows and
columns

Yes here it is.  I actually convert them all as strings, initially using  
options(stringsAsFactors=F) at the top of my code.

This what the initial dataframe looks like.  Please note this is a toy
dataset:

names    X    Y    Z    AorB
S1    BB    BB    BB    A
S2    AA    BB    BB    A
S3    AB    AB    AA    B
S4    AA    AA    NA    B


And the code to create this initial dataframe is:

names - c(S1, S2, S3, S4)
X - c(BB, AA, AB, AA)
Y - c(BB, BB, AB, AA)
Z - c(BB, BB, AA, NA)
AorB - c(A, A, B, B)

sample - data.frame(names, X, Y, Z, AorB)


The final data.frame should look like:

names  X    Y    Z    AorB
S1    0    0    0    A
S2    2    0    0    A
S3    1    1    0    B
S4    0    0    NA B

You're right! - I'll should be able to globally change all ABs and BAs to
1s. Thanks:)  I'm not exactly sure how to change AA and BB depending on AorB
for each row though.  Thoughts?

Thanks for your help thus far, David.

Best, Priya

On Wed, May 16, 2012 at 6:53 AM, David L Carlson dcarl...@tamu.edu wrote:
Can you show us what you want the final data.frame to look like? You've
created five variables stored as factors and you seem to be trying to change
those to numeric values? Is that correct?

Since AB and BA are always set to 1, you could just replace those values
globally rather than mess with the ifelse commands for those values. Only AA
and BB are affected by the value of AorB.

Your apply() function processes the data.frame by row so i is a vector
consisting of all the values in the row. You seem to be coding as if i was a
single integer (as in a for loop).

--
David L Carlson
Associate Professor of Anthropology
Texas AM University
College Station, TX 77843-4352


 -Original Message-
 From: r-help-boun...@r-project.org [mailto:r-help-bounces@r-
 project.org] On Behalf Of Priya Bhatt
 Sent: Wednesday, May 16, 2012 3:08 AM
 To: r-help@r-project.org
 Subject: [R] Help needed for efficient way to loop through rows and
 columns

 Dear R-helpers:

 I am trying to write a script that iterates through a dataframe that
 looks
 like this:


 Example dataset called sample:

 names - c(S1, S2, S3, S4)
 X - c(BB, AB, AB, AA)
 Y - c(BB, BB, AB, AA)
 Z - c(BB, BB, AB, NA)
 AorB - c(A, A, A, B)

 sample - data.frame(names, X, Y, Z, AorB)


 for a given row,

 if AorB == A, then AA == 2, AB = 1, BA = 1, BB = 0

 if AorB == B, then AA == 0, AB = 1, BA = 1, BB = 2

 I've been trying  to write this using apply and ifelse statements in
 hopes
 that my code runs quickly, but I'm afraid I've make a big mess.  See
 below:

 apply(sample, 1, function(i) {


   ifelse(sample$AorB[i] == A,
          (ifelse(sample[i,] == AA, sample[i,] - 2 ,
                  ifelse(sample[i,] == AB || sample[i,] == BA ,
 sample[i,] - 1,
                         ifelse(sample[i,] == BB, sample[i,] - 0,
 sample[i,] - NA )) )
           )   , ifelse(sample$AorB[i,] == B),
          (ifelse(sample[i,] == AA, sample[i,] - 0 ,
                  ifelse(sample[i,] == AB || sample[i,] == BA ,
 sample[i,] - 1,
                         ifelse(sample[i,] == BB, sample[i,] - 2,
 sample[i,] - NA) })


 Any Advice?

       [[alternative HTML version deleted]]

 __
 R-help@r-project.org mailing list
 https://stat.ethz.ch/mailman/listinfo/r-help
 PLEASE do read the posting guide http://www.R-project.org/posting-
 guide.html
 and provide commented, minimal, self-contained, reproducible code.

__
R-help@r-project.org mailing list
https://stat.ethz.ch/mailman/listinfo/r-help
PLEASE do read the posting guide http://www.R-project.org/posting-guide.html
and provide commented, minimal, self-contained, reproducible code.


Re: [R] Importing ASCII flat

2012-05-17 Thread David Winsemius


On May 17, 2012, at 8:17 AM, Richard Iles wrote:

I am trying to import Indian National Sample Survey Data. It is  
ASCII flat

and an example is below

 00146030602500111710107111201*01*00211 270104070204093
00246030602500111710107111201*02*00806104910519572 022   
2600

11503055   170  4005  00346030602500111710107111201*03*
111039204112  222

Previous post don't seem to provide much help. I have an document  
outlining

the layout of the data, but don't know how to start. An example of the
layout is that the values in bold indicate separate levels.Example of
layout is below:

Your data layout was mangled by passage of html formating through  
various mailers. Looking at:


http://mospi.nic.in/Mospi_New/site/inner.aspx?status=4menu_id=67

... I'm guessing you are showing only a tiny slice of the data  
specification.



 Sl.No. Item Blk Item Col Len Byte Position Remarks



1 Common Items  33 1  - 33 Auto-duplicated
2 Level  234  - 35  02 Generated
3 Filler 536  - 40 0 Generated
4 HHS Size 3 1 2  41  - 42
5 NIC Code(5-digit) 3 2 5 43  - 47
6 NCO Code(3-digit) 3 3 3 48  - 50
7 HHS type 3 4 1  51  -51
8 Religion 3 5 1  52  - 52


That was my best guess at what might have originally been the fixed  
format layout. You should look at:


?utils::read.fwf

If you can create a data.frame, dfrm,  from that data layout, then the  
most useful fields would be the names and the lengths. Perhaps this  
could work


read.fwf(file, widths=dfrm$Len, col.names= dfrm$Item)


As is obvious, I am relatively new to R.



Any constructive advice would be
welcome.


PLEASE, please, please : Learn to post in plain text.



Richard Iles
--
Economics Ph.D. student
Delhi School of Economics (visiting)
e: richard.i...@griffithuni.edu.au

website: https://sites.google.com/a/griffithuni.edu.au/richard-iles/home 
https://sites.google.com/site/richardileshome/home


[[alternative HTML version deleted]]

__
R-help@r-project.org mailing list
https://stat.ethz.ch/mailman/listinfo/r-help
PLEASE do read the posting guide http://www.R-project.org/posting-guide.html
and provide commented, minimal, self-contained, reproducible code.


David Winsemius, MD
West Hartford, CT

__
R-help@r-project.org mailing list
https://stat.ethz.ch/mailman/listinfo/r-help
PLEASE do read the posting guide http://www.R-project.org/posting-guide.html
and provide commented, minimal, self-contained, reproducible code.


[R] nls and if statements

2012-05-17 Thread DWatts
Hi All,

I have a situation where I want an 'if' variable to be parameterized. It's
entirely possible that the way I'm trying to do this is wrong, especially
given the error message I get that indicates I can't do this using an 'if'
statement. 

Essentially, I have data where I think a relationship enters when a variable
(here Pwd) is below some value (z). I don't know that value, so I want to
fit it. The data is Pw, Tsoil, and Cfl. 

nlstest=nls(if(Pwz){Cfl~K*exp(-b*Pw)+c
}else{
Cfl~K*(exp(-a*Tsoil))*exp(-b*Pw)+c
},start=c(K=5.5, a=0.1, b=0.1, c=2, z=5))

Which returns the error:
Error in inherits(object, formula) : object 'z' not found

Is there a better way to try and allow a conditional variable to, well,
vary?

--
View this message in context: 
http://r.789695.n4.nabble.com/nls-and-if-statements-tp4630391.html
Sent from the R help mailing list archive at Nabble.com.

__
R-help@r-project.org mailing list
https://stat.ethz.ch/mailman/listinfo/r-help
PLEASE do read the posting guide http://www.R-project.org/posting-guide.html
and provide commented, minimal, self-contained, reproducible code.


Re: [R] kolmogorov-Smirnov critical values

2012-05-17 Thread aramos
Thank you all for your help!! !

Alex

--
View this message in context: 
http://r.789695.n4.nabble.com/kolmogorov-Smirnov-critical-values-tp4630245p4630393.html
Sent from the R help mailing list archive at Nabble.com.

__
R-help@r-project.org mailing list
https://stat.ethz.ch/mailman/listinfo/r-help
PLEASE do read the posting guide http://www.R-project.org/posting-guide.html
and provide commented, minimal, self-contained, reproducible code.


Re: [R] Correlation Matrix

2012-05-17 Thread Cren
Hi,

unless you're dealing with heteroskedastic datas, the command *cor(x)* will
be enough, where *x* is your data matrix; in this function you can easily
select the method which has to be used: Pearson's, Kendall's or Spearman's
correlation.

--
View this message in context: 
http://r.789695.n4.nabble.com/Correlation-Matrix-tp4630389p4630392.html
Sent from the R help mailing list archive at Nabble.com.

__
R-help@r-project.org mailing list
https://stat.ethz.ch/mailman/listinfo/r-help
PLEASE do read the posting guide http://www.R-project.org/posting-guide.html
and provide commented, minimal, self-contained, reproducible code.


Re: [R] how to download source code, pdf manual for any package from internet?

2012-05-17 Thread Ben Bolker
sagarnikam123 sagarnikam123 at gmail.com writes:

 
 i can get list of available packages by
  r-available.packages()
 
 No of available packages
  length(r[,1])
 [1] 3739
 
 single package name
  r[1,1]
 [1] ACCLMA
 
 i want download all packages  starting with A / a alphabets,
 with their source code, pdf manual/vignettes from internet
 
 --
 View this message in context:
http://r.789695.n4.nabble.com/how-to-download-source-code-pdf-manual-for-any-package-from-internet-tp4630354.html
 Sent from the R help mailing list archive at Nabble.com.
 
 

grep(^[Aa],rownames(r),value=TRUE)

gets you the names of the 143 packages starting with A or a.

If you google ACCLMA you will find

http://cran.r-project.org/web/packages/ACCLMA/

with further links

http://cran.r-project.org/src/contrib/ACCLMA_1.0.tar.gz
http://cran.r-project.org/web/packages/ACCLMA/ACCLMA.pdf

You can use paste() and download.file() in a loop to get all of the
packages.

ACCLMA happens not to have any vignettes.  It's a little hard for me
to see how you can figure out the URLs of package-associated vignettes,
without web-scraping (which shouldn't actually be too hard, but I won't
illustrate here how to do it)

  You probably want to use the URL of your local mirror rather than
the main CRAN site when doing this.

  Ben Bolker

__
R-help@r-project.org mailing list
https://stat.ethz.ch/mailman/listinfo/r-help
PLEASE do read the posting guide http://www.R-project.org/posting-guide.html
and provide commented, minimal, self-contained, reproducible code.


Re: [R] ctree for suvival analysis problem

2012-05-17 Thread ofraam
Thanks very much Achim!
I was indeed using pec which requires rms... 

I see now that the old version of pec that I used didn't require rms and
therefore I had no problem.. I'm guessing I don't need the rms methods for
my use of pec, but am not sure how I can remove the dependency or
alternatively mask back the Surv object to survival package. Any ideas for
that?

Thanks again!

Ofra


--
View this message in context: 
http://r.789695.n4.nabble.com/ctree-for-suvival-analysis-problem-tp4630362p4630403.html
Sent from the R help mailing list archive at Nabble.com.

__
R-help@r-project.org mailing list
https://stat.ethz.ch/mailman/listinfo/r-help
PLEASE do read the posting guide http://www.R-project.org/posting-guide.html
and provide commented, minimal, self-contained, reproducible code.


Re: [R] nls and if statements

2012-05-17 Thread David Winsemius


On May 17, 2012, at 10:06 AM, DWatts wrote:


Hi All,

I have a situation where I want an 'if' variable to be  
parameterized. It's
entirely possible that the way I'm trying to do this is wrong,  
especially
given the error message I get that indicates I can't do this using  
an 'if'

statement.

Essentially, I have data where I think a relationship enters when a  
variable
(here Pwd) is below some value (z). I don't know that value, so I  
want to

fit it. The data is Pw, Tsoil, and Cfl.

nlstest=nls(if(Pwz){Cfl~K*exp(-b*Pw)+c
}else{
Cfl~K*(exp(-a*Tsoil))*exp(-b*Pw)+c
},start=c(K=5.5, a=0.1, b=0.1, c=2, z=5))



Perhaps recasting as Boolean equivalent:

nlstest== - nls({Cfl~ (Pwz)*K*exp(-b*Pw)+c) +
   !(Pwz)*(K*(exp(-a*Tsoil))*exp(-b*Pw)+c)},
 start=c(K=5.5, a=0.1, b=0.1, c=2, z=5))


and provide commented, minimal, self-contained, reproducible code.


Untested in absence of data.

--

David Winsemius, MD
West Hartford, CT

__
R-help@r-project.org mailing list
https://stat.ethz.ch/mailman/listinfo/r-help
PLEASE do read the posting guide http://www.R-project.org/posting-guide.html
and provide commented, minimal, self-contained, reproducible code.


Re: [R] nls and if statements

2012-05-17 Thread David Winsemius


On May 17, 2012, at 11:08 AM, David Winsemius wrote:



On May 17, 2012, at 10:06 AM, DWatts wrote:


Hi All,

I have a situation where I want an 'if' variable to be  
parameterized. It's
entirely possible that the way I'm trying to do this is wrong,  
especially
given the error message I get that indicates I can't do this using  
an 'if'

statement.

Essentially, I have data where I think a relationship enters when a  
variable
(here Pwd) is below some value (z). I don't know that value, so I  
want to

fit it. The data is Pw, Tsoil, and Cfl.

nlstest=nls(if(Pwz){Cfl~K*exp(-b*Pw)+c
}else{
Cfl~K*(exp(-a*Tsoil))*exp(-b*Pw)+c
},start=c(K=5.5, a=0.1, b=0.1, c=2, z=5))



Perhaps recasting as Boolean equivalent:

nlstest== - nls({Cfl~ (Pwz)*K*exp(-b*Pw)+c) +

#needs another open-paren  ^

  !(Pwz)*(K*(exp(-a*Tsoil))*exp(-b*Pw)+c)},
start=c(K=5.5, a=0.1, b=0.1, c=2, z=5))



nlstest== - nls({Cfl~ (Pwz)*( K*exp(-b*Pw)+c) +
  !(Pwz)*(K*(exp(-a*Tsoil))*exp(-b*Pw)+c)},
start=c(K=5.5, a=0.1, b=0.1, c=2, z=5))




and provide commented, minimal, self-contained, reproducible code.



Still untested in absence of data.


--


David Winsemius, MD
West Hartford, CT

__
R-help@r-project.org mailing list
https://stat.ethz.ch/mailman/listinfo/r-help
PLEASE do read the posting guide http://www.R-project.org/posting-guide.html
and provide commented, minimal, self-contained, reproducible code.


Re: [R] ctree for suvival analysis problem

2012-05-17 Thread David Winsemius


On May 17, 2012, at 11:04 AM, ofraam wrote:


Thanks very much Achim!
I was indeed using pec which requires rms...

I see now that the old version of pec that I used didn't require rms  
and
therefore I had no problem.. I'm guessing I don't need the rms  
methods for

my use of pec, but am not sure how I can remove the dependency or
alternatively mask back the Surv object to survival package. Any  
ideas for

that?


Perhaps:

GBSG2ct - ctree(survival::Surv(time, cens) ~ .,data = GBSG2)

--
David Winsemius, MD
West Hartford, CT

__
R-help@r-project.org mailing list
https://stat.ethz.ch/mailman/listinfo/r-help
PLEASE do read the posting guide http://www.R-project.org/posting-guide.html
and provide commented, minimal, self-contained, reproducible code.


Re: [R] ctree for suvival analysis problem

2012-05-17 Thread ofraam
Thanks David!!!
Sorry for the stupid question...

Seems like I still have issues with pec, but I will work on it some more...

--
View this message in context: 
http://r.789695.n4.nabble.com/ctree-for-suvival-analysis-problem-tp4630362p4630408.html
Sent from the R help mailing list archive at Nabble.com.

__
R-help@r-project.org mailing list
https://stat.ethz.ch/mailman/listinfo/r-help
PLEASE do read the posting guide http://www.R-project.org/posting-guide.html
and provide commented, minimal, self-contained, reproducible code.


Re: [R] how to find outliers from the list of values

2012-05-17 Thread Bert Gunter
Petr et. al:

FWIW (probably not much).

As you know, tens of thousands of pages about outliers have been
written by statisticians. IMHO, it is another of the really terrible
ideas of our discipline and has led to much scientific abuse, as
indicated by this posting. For this reason, I have eliminated it from
my vocabulary, using instead unusual or unexpected values, whose
meaning and purpose is pretty much as you described -- to bring the
user's attention to data issues that may require investigation and
intervention.

By eliminating the term, I feel it excises the notion that there can
somehow be statistical tests (alone) that can,  irrespective of
scientific context, statistically identify illegitimate data. A
really dangerous and pernicious idea imho.

Best,
Bert

On Thu, May 17, 2012 at 6:44 AM, Petr PIKAL petr.pi...@precheza.cz wrote:
 Hi

 I had not see any answer yet but maybe there is nobody who wants to touch
 the elusive object of outlier. Neither me, but here are some ideas how
 one can proceed.

 First of all its always up to you what is considered an outlier and how
 will you deal with them.

 I usually call an outlier any item which does not fit to the pattern and
 the pattern is usually best observed by some plotting function. You can
 identify outlier points, inspect the data source, correct typing mistakes
 and only if the value is really measured and you can not find any reason
 why it has such value it is real outlier. Then ***you*** need to decide
 what to do with it - discard, can come from some long tailed distribution,
 ...

 So here are my 0.02$ regarding an outlier theme.

 Regards
 Petr


 Hi,
         I am new to R and I would like to get your help in finding
 'outliers'.
  I have mvoutlier package installed in my system and added the package .
 But I not able find a function from 'mvoutlier' package which will
 identify
 'outliers'.
 This is the sample list of data I have got which has one out-lier.
    11489  11008  11873  8000  9558  8645  8024  8371  It will be of
 great help if somebody have got an example script for the same.

 Thanks  Regards,
 Thomas

    [[alternative HTML version deleted]]

 __
 R-help@r-project.org mailing list
 https://stat.ethz.ch/mailman/listinfo/r-help
 PLEASE do read the posting guide
 http://www.R-project.org/posting-guide.html
 and provide commented, minimal, self-contained, reproducible code.

 __
 R-help@r-project.org mailing list
 https://stat.ethz.ch/mailman/listinfo/r-help
 PLEASE do read the posting guide http://www.R-project.org/posting-guide.html
 and provide commented, minimal, self-contained, reproducible code.



-- 

Bert Gunter
Genentech Nonclinical Biostatistics

Internal Contact Info:
Phone: 467-7374
Website:
http://pharmadevelopment.roche.com/index/pdb/pdb-functional-groups/pdb-biostatistics/pdb-ncb-home.htm

__
R-help@r-project.org mailing list
https://stat.ethz.ch/mailman/listinfo/r-help
PLEASE do read the posting guide http://www.R-project.org/posting-guide.html
and provide commented, minimal, self-contained, reproducible code.


[R] Covariance structure for lme

2012-05-17 Thread Charles Determan Jr
Greetings again R users,

Some of you will likely recognize me but I hope you can help me once
more.  I have tried the mixed model mailing list for this question but have
yet to find a solution.  As such I hope someone will have another idea.

I have previously attempted to replicate the UN, CS, and AR(1)
covariance structures used in SAS PROC MIXED.  However, my efforts
have fallen short on replicating the Variance Components (VC)
structure.  I have read that it is also known as a diagonal structure.
 Below I have copied over all the models I have tried and their output
with no success.  Perhaps someone here will see my error or something
I have overlooked.  I have attached the data for this particular
model.  Thanks to all, I certainly cannot thank this help list enough.
 I you need any further information/clarification, please ask.
Cheers, Charles

dat=read.table(C:/subset.csv,sep=,,header=TRUE, na.strings=.)
attach(dat)

dat34=dat[Group %in% c(3, 4),]
attach(dat34)
liver34=within(dat34, {
   Group=factor(Group)
   Event_name=factor(Event_name)
   Died=factor(Died)
   ID=factor(ID)
})
attach(liver34)

contrasts(liver34$Old_Event_name)=contr.sum(n=6)
contrasts(liver34$Pig_group)=contr.sum(n=2)
contrasts(liver34$Died)=contr.sum(n=2)

##What is should be from SAS
#CV var
#Type 3 Tests of Fixed Effects
#Effect  NumDF DenDF F Value Pr  F
#Group   1  22  0.250.6244
#Died1  22  6.550.0179
#Group*Died  1  22  4.430.0470

fit.1=lme(var~Group*Died,
   random=~1|ID,
   data=dat34)
anova(fit.1, type=marginal, adjustSigma=F)
#   numDF denDF   F-value p-value
#(Intercept)1   101 227.58700  .0001
#Group  122   0.18320  0.6728
#Died   122   3.63388  0.0698
#Group:Died 122   3.04103  0.0951

fit.2=lme(var~Group*Died,
   data=dat34,
   random=~1|ID/Died)
anova(fit.2, type=marginal, adjustSigma=F)
#   numDF denDF  F-value p-value
#(Intercept)1   101 77.99004  .0001
#Group  122  1.46275  0.2393
#Died   122  5.84535  0.0243
#Group:Died 122  3.04103  0.0951

fit.3=lme(var~Group*Died,
   random=list(ID=pdSymm(~Event_name)),
   data=dat34)
anova(fit.3, type=marginal, adjustSigma=F)
#   numDF denDF   F-value p-value
#(Intercept)1   101 273.10918  .0001
#Group  122   0.69692  0.4128
#Died   122   1.43316  0.2440
#Group:Died 122   5.74399  0.0255

fit.4=lme(var~Group*Died,
   random=list(ID=pdSymm(~Group)),
   data=dat34)
anova(fit.4, type=marginal, adjustSigma=F)
#   numDF denDF   F-value p-value
#(Intercept)1   101 235.13889  .0001
#Group  122   0.15878  0.6941
#Died   122   3.83253  0.0631
#Group:Died 122   3.01222  0.0966

fit.5=lme(var~Group*Died,
   random=list(ID=pdSymm(~Group)),
   data=dat34,
   weights=varIdent(form=~1|Event_name))
anova(fit.5, type=marginal, adjustSigma=F)
#   numDF denDF   F-value p-value
#(Intercept)1   101 277.16705  .0001
#Group  122   0.23901  0.6298
#Died   122   3.99283  0.0582
#Group:Died 122   3.23135  0.0860

fit.6=lme(var~Group*Died,
   random=list(ID=pdSymm(~Group)),
   data=dat34,
   weights=varIdent(form=~1|Event_name))
anova(fit.6, type=marginal, adjustSigma=F)
#   numDF denDF   F-value p-value
#(Intercept)1   101 277.16705  .0001
#Group  122   0.23901  0.6298
#Died   122   3.99283  0.0582
#Group:Died 122   3.23135  0.0860

fit.7=lme(var~(Group*Died),
   random=list(ID=pdCompSymm(~Died)),
   data=dat34)
anova(fit.7, type=marginal, adjustSigma=F)
#   numDF denDF  F-value p-value
#(Intercept)1   101 85.83799  .0001
#Group  122  1.60624  0.2183
#Died   122  4.71795  0.0409
#Group:Died 122  2.65379  0.1175

fit.8=lme(var~(Group*Died),
   data=dat34,
   random=~1|ID,
   corr=corSymm())
anova(fit.8, type=marginal, adjustSigma=F)
#   numDF denDF   F-value p-value
#(Intercept)1   101 119.54403  .0001
#Group  122   4.58972  0.0435
#Died   122   8.01715  0.0097
#Group:Died 122   5.27470  0.0315

fit.9=lme(var~(Group*Died),
   data=dat34,
   random=list(ID=pdDiag(~Group*Died)),
   corr=corSymm(, ~1|ID))
#  Error in lme.formula(var ~ (Group * Died), data = dat34, random =
list(ID = pdDiag(~Group *  :
#  nlminb problem, convergence error code = 1
#  message = iteration limit reached without convergence (9)

fit.10=lme(var~(Group*Died),
   data=dat34,
   random=list(ID=pdDiag(~Group*Died)),
   corr=NULL)
anova(fit.10, type=marginal, adjustSigma=F)
#   numDF denDF  F-value p-value
#(Intercept)1   101 93.90211  .0001
#Group  122  1.75311  0.1991
#Died   122  6.84379  0.0158

Re: [R] how to find outliers from the list of values

2012-05-17 Thread Robert Baer
Petr and Bert offer sound advice. At the risk of getting completely 
ostracized, here's how you could find outliers using the definition of 
'outlier' used by R's boxplot function and at the same time see your data.



dat = c(11489,  11008,  11873,  8000,  9558,  8645,  8024,  8371)
a = boxplot(dat)
a$out

[1] 8e+07



-Original Message- 
From: Prakash Thomas

Sent: Tuesday, May 15, 2012 7:00 AM
To: r-help@r-project.org
Subject: [R] how to find outliers from the list of values

Hi,
   I am new to R and I would like to get your help in finding
'outliers'.
I have mvoutlier package installed in my system and added the package .
But I not able find a function from 'mvoutlier' package which will identify
'outliers'.
This is the sample list of data I have got which has one out-lier.
  11489  11008  11873  8000  9558  8645  8024  8371  It will be of
great help if somebody have got an example script for the same.

Thanks  Regards,
Thomas

[[alternative HTML version deleted]]

__
R-help@r-project.org mailing list
https://stat.ethz.ch/mailman/listinfo/r-help
PLEASE do read the posting guide http://www.R-project.org/posting-guide.html
and provide commented, minimal, self-contained, reproducible code.


--
Robert W. Baer, Ph.D.
Professor of Physiology
Kirksville College of Osteopathic Medicine
A. T. Still University of Health Sciences
800 W. Jefferson St.
Kirksville, MO 63501
660-626-2322
FAX 660-626-2965 


__
R-help@r-project.org mailing list
https://stat.ethz.ch/mailman/listinfo/r-help
PLEASE do read the posting guide http://www.R-project.org/posting-guide.html
and provide commented, minimal, self-contained, reproducible code.


Re: [R] nls and if statements

2012-05-17 Thread DWatts

David Winsemius wrote
 
 and provide commented, minimal, self-contained, reproducible code.

 Still untested in absence of data.
 

Thank you, and my apologies for not giving some sample numbers. Below is a
short subset of the data set I'm working with.

Cfl=c(2.61,4.21,2,2.75,7.47,1.2,3.24,12.49,2.37,3.28,4.3,2.61,2.75,2.92,3.78,2.25,2.84,3.33,2.39)
Tsoil=c(30.01,27.72,34.91,21.96,22.83,29.65,20.56,23.72,33.96,31.57,19.15,28.49,30.02,20.14,30.23,33.99,32.58,17.87,19.78)
Pw=c(30,29,24,22,22,41.2,-33.1,-49.21,29.6,24.5,-6.1544,42.8,63,55.5,43.3,38.5,87.2,-0.32,79.5)

Rui's code seems to elegantly do what I was attempting (and will try the
Boolian version next), but I'm having initial parameterization problems.
I've tried doing what I've done before, which is use the excel solver to
give me some simple early estimates of params, but it doesn't appear to be
helping in this case. 

nlstest=nls(Cfl~K*ifelse(Pw  z, 1, exp(-a*Tsoil))*exp(-b*Pw)+c,
start=c(K=0.14, a=0.01, b=0.04, c=3.5, z=5), trace=T) 

I'll keep chugging away at this. Thank you for your help!



--
View this message in context: 
http://r.789695.n4.nabble.com/nls-and-if-statements-tp4630391p4630410.html
Sent from the R help mailing list archive at Nabble.com.

__
R-help@r-project.org mailing list
https://stat.ethz.ch/mailman/listinfo/r-help
PLEASE do read the posting guide http://www.R-project.org/posting-guide.html
and provide commented, minimal, self-contained, reproducible code.


[R] Correlation Matrix

2012-05-17 Thread mahdi
Could anyone can help me telling the way how I can build correlation matrix
in R? Thanks in advance.

--
View this message in context: 
http://r.789695.n4.nabble.com/Correlation-Matrix-tp4630389.html
Sent from the R help mailing list archive at Nabble.com.

__
R-help@r-project.org mailing list
https://stat.ethz.ch/mailman/listinfo/r-help
PLEASE do read the posting guide http://www.R-project.org/posting-guide.html
and provide commented, minimal, self-contained, reproducible code.


Re: [R] Correlation Matrix

2012-05-17 Thread mahdi
Thanks a lot. Suppose I want to use Pearson's method, then what I have to do? 

--
View this message in context: 
http://r.789695.n4.nabble.com/Correlation-Matrix-tp4630389p4630396.html
Sent from the R help mailing list archive at Nabble.com.

__
R-help@r-project.org mailing list
https://stat.ethz.ch/mailman/listinfo/r-help
PLEASE do read the posting guide http://www.R-project.org/posting-guide.html
and provide commented, minimal, self-contained, reproducible code.


Re: [R] nls and if statements

2012-05-17 Thread Rui Barradas
Hello,

I don't know if this is what you want, but your formula is equivalent to

nlstest=nls(Cfl~K*ifelse(Pw  z, 1, exp(-a*Tsoil))*exp(-b*Pw)+c
,start=c(K=5.5, a=0.1, b=0.1, c=2, z=5)) 

Hope this helps,

Rui Barradas


--
View this message in context: 
http://r.789695.n4.nabble.com/nls-and-if-statements-tp4630391p4630398.html
Sent from the R help mailing list archive at Nabble.com.

__
R-help@r-project.org mailing list
https://stat.ethz.ch/mailman/listinfo/r-help
PLEASE do read the posting guide http://www.R-project.org/posting-guide.html
and provide commented, minimal, self-contained, reproducible code.


Re: [R] Correlation Matrix

2012-05-17 Thread Joshua Wiley
Hi Mahdi,

Look at the documentation for cor(), by typing ?cor or help(cor).
Pearson is the default and it is trivial to select the others.  I
suggest you try searching google or reading R's documentation before
posting to the list.  You may not understand it all, but it shows you
tried to work on your own and let's you ask more specific questions.
For example: I am using function foo(), and read the documentation,
but was confused by this paragraph some paragraph from docs for foo.
 Then we can come along and so oh, what that means is _.

Cheers,

Josh

On Thu, May 17, 2012 at 7:22 AM, mahdi mahd...@gmail.com wrote:
 Thanks a lot. Suppose I want to use Pearson's method, then what I have to do?

 --
 View this message in context: 
 http://r.789695.n4.nabble.com/Correlation-Matrix-tp4630389p4630396.html
 Sent from the R help mailing list archive at Nabble.com.

 __
 R-help@r-project.org mailing list
 https://stat.ethz.ch/mailman/listinfo/r-help
 PLEASE do read the posting guide http://www.R-project.org/posting-guide.html
 and provide commented, minimal, self-contained, reproducible code.



-- 
Joshua Wiley
Ph.D. Student, Health Psychology
Programmer Analyst II, Statistical Consulting Group
University of California, Los Angeles
https://joshuawiley.com/

__
R-help@r-project.org mailing list
https://stat.ethz.ch/mailman/listinfo/r-help
PLEASE do read the posting guide http://www.R-project.org/posting-guide.html
and provide commented, minimal, self-contained, reproducible code.


Re: [R] Correlation Matrix

2012-05-17 Thread Bert Gunter
Josh:

A very nice, clear, polite, concise, and reasoned alternative to RTFM !

Probably should be templated somehow, given the volume of queries of
this sort that this list receives. (The posting guide is too involved
to serve in its stead.)

Cheers,
Bert



On Thu, May 17, 2012 at 9:31 AM, Joshua Wiley jwiley.ps...@gmail.com wrote:
 Hi Mahdi,

 Look at the documentation for cor(), by typing ?cor or help(cor).
 Pearson is the default and it is trivial to select the others.  I
 suggest you try searching google or reading R's documentation before
 posting to the list.  You may not understand it all, but it shows you
 tried to work on your own and let's you ask more specific questions.
 For example: I am using function foo(), and read the documentation,
 but was confused by this paragraph some paragraph from docs for foo.
  Then we can come along and so oh, what that means is _.

 Cheers,

 Josh

 On Thu, May 17, 2012 at 7:22 AM, mahdi mahd...@gmail.com wrote:
 Thanks a lot. Suppose I want to use Pearson's method, then what I have to do?

 --
 View this message in context: 
 http://r.789695.n4.nabble.com/Correlation-Matrix-tp4630389p4630396.html
 Sent from the R help mailing list archive at Nabble.com.

 __
 R-help@r-project.org mailing list
 https://stat.ethz.ch/mailman/listinfo/r-help
 PLEASE do read the posting guide http://www.R-project.org/posting-guide.html
 and provide commented, minimal, self-contained, reproducible code.



 --
 Joshua Wiley
 Ph.D. Student, Health Psychology
 Programmer Analyst II, Statistical Consulting Group
 University of California, Los Angeles
 https://joshuawiley.com/

 __
 R-help@r-project.org mailing list
 https://stat.ethz.ch/mailman/listinfo/r-help
 PLEASE do read the posting guide http://www.R-project.org/posting-guide.html
 and provide commented, minimal, self-contained, reproducible code.



-- 

Bert Gunter
Genentech Nonclinical Biostatistics

Internal Contact Info:
Phone: 467-7374
Website:
http://pharmadevelopment.roche.com/index/pdb/pdb-functional-groups/pdb-biostatistics/pdb-ncb-home.htm

__
R-help@r-project.org mailing list
https://stat.ethz.ch/mailman/listinfo/r-help
PLEASE do read the posting guide http://www.R-project.org/posting-guide.html
and provide commented, minimal, self-contained, reproducible code.


[R] Windows Task Scheduler and R updates. Need basic tips

2012-05-17 Thread Paul Johnson
This is a basic Windows system administrator problem, asked by a Linux
guy who is helping out in a Windows lab.

I want to keep R packages up to date on MS Windows 7 with a job in the
Task Scheduler.  I have an R program that I can run (as
administrator) that updates the existing packages and then installs
all the new ones.

I do not understand how to run that in a dependable way in the scheduler.

If I put the update script R-update.R in, for example, in

C:\Program Files\R\R-update.R

Then what?  Do I need a CMD batch script to run the R script?

I can't tell where Windows wants to write the standard output and
error for my R job.

And while I'm asking, does Windows care if I run

R CMD BATCH C:\Program Files\R\R-update.R

or

R --vanilla -f C:\Program Files\R\R-update.R

pj
-- 
Paul E. Johnson
Professor, Political Science    Assoc. Director
1541 Lilac Lane, Room 504     Center for Research Methods
University of Kansas               University of Kansas
http://pj.freefaculty.org            http://quant.ku.edu

__
R-help@r-project.org mailing list
https://stat.ethz.ch/mailman/listinfo/r-help
PLEASE do read the posting guide http://www.R-project.org/posting-guide.html
and provide commented, minimal, self-contained, reproducible code.


Re: [R] nls and if statements

2012-05-17 Thread Peter Ehlers

On 2012-05-17 08:40, DWatts wrote:


David Winsemius wrote



and provide commented, minimal, self-contained, reproducible code.



Still untested in absence of data.



Thank you, and my apologies for not giving some sample numbers. Below is a
short subset of the data set I'm working with.

Cfl=c(2.61,4.21,2,2.75,7.47,1.2,3.24,12.49,2.37,3.28,4.3,2.61,2.75,2.92,3.78,2.25,2.84,3.33,2.39)
Tsoil=c(30.01,27.72,34.91,21.96,22.83,29.65,20.56,23.72,33.96,31.57,19.15,28.49,30.02,20.14,30.23,33.99,32.58,17.87,19.78)
Pw=c(30,29,24,22,22,41.2,-33.1,-49.21,29.6,24.5,-6.1544,42.8,63,55.5,43.3,38.5,87.2,-0.32,79.5)

Rui's code seems to elegantly do what I was attempting (and will try the
Boolian version next), but I'm having initial parameterization problems.
I've tried doing what I've done before, which is use the excel solver to
give me some simple early estimates of params, but it doesn't appear to be
helping in this case.

nlstest=nls(Cfl~K*ifelse(Pw  z, 1, exp(-a*Tsoil))*exp(-b*Pw)+c,
start=c(K=0.14, a=0.01, b=0.04, c=3.5, z=5), trace=T)

I'll keep chugging away at this. Thank you for your help!



If the above data is all you have, I wouldn't chugg too long.
Looking at plots of Cfl~Pw and Cfl~Tsoil says to me that you
have far too much variability to fit your model.

You might find this plot instructive:

  library(rgl)
  plot3d(Pw, Tsoil, Cfl, col=1+(Pw5), size=5)


Peter Ehlers

__
R-help@r-project.org mailing list
https://stat.ethz.ch/mailman/listinfo/r-help
PLEASE do read the posting guide http://www.R-project.org/posting-guide.html
and provide commented, minimal, self-contained, reproducible code.


Re: [R] Complex sort problem

2012-05-17 Thread Petr Savicky
On Thu, May 17, 2012 at 06:45:52AM -0400, Axel Urbiz wrote:
 Dear List,
 
 Is there a way I can sort a sample based on a sort index constructed from
 the data from which the sample is taken? Basically, I need to take 'many'
 samples from the same source data and sort them. This can be very time
 consuming for long vectors. Is there any way I can sort the data only once
 initially, and use that sort order for the samples?
 
 I believe that idea is what is implemented in tree-based classifiers, so
 the data is sorted only once initially and that sort order is used for the
 child nodes.
 
 
 set.seed(12345)
 x - sample(0:100, 10)
 x.order - order(x)
 x.sorted - x[x.order]
 
 sample.ind - sample(1:length(x), 5, replace = TRUE)  #sample 1/2 size with 
 replacement
 x.sample - x[sample.ind]
 
 x.sample.sorted -   #??? (without sorting again)

Hi.

Formally, it is possible to avoid sorting using tabulate() and rep().
However, i am not sure, whether this approach is more efficient.

  set.seed(12345)
  x - sample(0:100, 10)
  x.order - order(x)
  x.sorted - x[x.order]
 
  sample.ind - sample(1:length(x), 5, replace = TRUE)  #sample 1/2 size with 
replacement
 
  x.sample - x.sorted[sample.ind]
  freq - tabulate(sample.ind, nbins=length(x))
  x.sample.sorted - rep(x.sorted, times=freq)

  identical(sort(x.sample), x.sample.sorted) # [1] TRUE

Note that x.sample is created from x.sorted in order to make x.sample
and x.sample.sorted consistent. Since sample.ind has random order, the
distributions of x[sample.ind] and x.sorted[sample.ind] are the same.

Computing the frequencies of indices, whose range is known in advance,
can be done in linear time, so theoretically more efficiently than
sorting. However, only a test may determine, what is more efficient in
your situation.

Hope this helps.

Petr Savicky.

__
R-help@r-project.org mailing list
https://stat.ethz.ch/mailman/listinfo/r-help
PLEASE do read the posting guide http://www.R-project.org/posting-guide.html
and provide commented, minimal, self-contained, reproducible code.


Re: [R] hmd.mx

2012-05-17 Thread MacQueen, Don
Type
  str(slovenia)
or perhaps
  names(slovenia)
to find out more about how the slovenia object is structured, and the
correct names of its elements.
Also 
  ?hmd.mx
(also see,for example,
http://help.pop.psu.edu/help-by-software-package/r-project/demography-in-r/
demography-package-for-r)

Also, since hmd.mx is not part of base R, please identify what package it
is from.

The error message typically comes from converting character to numeric
when some of the values are not numeric. Try, for example,
  as.numeric( c('1','2','A','4') )

-Don


-- 
Don MacQueen

Lawrence Livermore National Laboratory
7000 East Ave., L-627
Livermore, CA 94550
925-423-1062





On 5/17/12 2:41 AM, krmartina martina.kruec...@gmail.com wrote:

I have trouble using function hmd.mx
I have all packages required.
I call function:

slovenia - hmd.mx(SVN, username, password, Slovenia) :

and I get this error:
  NAs introduced by coercion

And if I then call slovenia, I get:

 slovenia
Mortality data for Slovenia
Series: female male total
Years: 1983 - 2009
Ages:  0 - 110

But I do not get any value (mortality, ...):
 slovenia$ages
NULL
 slovenia$years
NULL

I tried also for two different countries and I got the same result.

Can please anybody help me with this problem?

Thank you very much.
Martina






--
View this message in context:
http://r.789695.n4.nabble.com/hmd-mx-tp4630350.html
Sent from the R help mailing list archive at Nabble.com.

__
R-help@r-project.org mailing list
https://stat.ethz.ch/mailman/listinfo/r-help
PLEASE do read the posting guide
http://www.R-project.org/posting-guide.html
and provide commented, minimal, self-contained, reproducible code.

__
R-help@r-project.org mailing list
https://stat.ethz.ch/mailman/listinfo/r-help
PLEASE do read the posting guide http://www.R-project.org/posting-guide.html
and provide commented, minimal, self-contained, reproducible code.


Re: [R] caret: Error when using rpart and CV != LOOCV

2012-05-17 Thread Max Kuhn
Dominik,

There are a number of formulations of this statistic (see the
Kvålseth[*] reference below).

I tend to think of R^2 as the proportion of variance explained by the
model[**]. With the traditional formula, it is possible to get
negative proportions (if there are extreme outliers in the
predictions, the negative proportion can be very large). I used this
formulation because it is always on (0, 1). It is called R^2 after
all!

Here is an example:

 set.seed(1)
 simObserved - rnorm(100)
 simPredicted - simObserved + rnorm(100)*.1

 cor(simObserved, simPredicted)^2
[1] 0.9887525
 customSummary(data.frame(obs = simObserved,
+  pred = simPredicted))
  RMSE   Rsquared
0.09538273 0.98860908

 simPredicted[1]
[1] -0.6884905
 simPredicted[1] - 10

 cor(simObserved, simPredicted)^2
[1] 0.3669257
 customSummary(data.frame(obs = simObserved,
+  pred = simPredicted))
 RMSE  Rsquared
 1.066900 -0.425169

It is somewhat extreme, but it does happen.

Max


* Kvålseth, T. (1985). Cautionary note about $R^2$. American
statistician, 39(4), 279–285.
* This is a very controversial statement when non-linear models are
used. I'd rather use RMSE, but many scientists I work with still think
in terms of R^2 regardless of the model. The randomForest function
also computes this statistic, but calls it % Var explained instead
of explicitly labeling it as R^2. This statistic has generated
heated debates and I hope that I will not have to wear a scarlet R in
Nashville in a few weeks.


On Thu, May 17, 2012 at 1:35 PM, Dominik Bruhn domi...@dbruhn.de wrote:
 Hy Max,
 thanks again for the answer.

 I checked the caret implementation and you were right. If the
 predictions for the model constant (or sd(pred)==0) then the
 implementation returns a NA for the rSquare (in postResample). This is
 mainly because the caret implementation uses `cor` (from the
 stats-package) which would throw a error for values with sd(pred)==0.

 Do you know why this is implemented in this way? I wrote my own
 summaryFunction which calculates rSquare by hand and it works fine. It
 nevertheless does NOT(!) generate the same values as the original
 implementation. It seems that the calcuation of Rsquare does not seem to
 be consistent. I took mine from Wikipedia [1].

 Here is my code:
 ---
 customSummary - function (data, lev = NULL, model = NULL) {
         #Calulate rSquare
         ssTot - sum((data$obs-mean(data$obs))^2)
         ssErr - sum((data$obs-data$pred)^2)
         rSquare - 1-(ssErr/ssTot)

         #Calculate MSE
         mse - mean((data$pred - data$obs)^2)

         #Aggregate
         out - c(sqrt(mse), 1-(ssErr/ssTot))
         names(out) - c(RMSE, Rsquared)

         return(out)
 }
 ---

 [1]: http://en.wikipedia.org/wiki/Coefficient_of_determination#Definitions

 Thanks!
 Dominik




 On 17/05/12 04:10, Max Kuhn wrote:
 Dominik,

 See this line:

   Min. 1st Qu.  Median    Mean 3rd Qu.    Max.
  30.37   30.37   30.37   30.37   30.37   30.37

 The variance of the predictions is zero. caret uses the formula for
 R^2 by calculating the correlation between the observed data and the
 predictions which uses sd(pred) which is zero. I believe that the same
 would occur with other formulas for R^2.

 Max

 On Wed, May 16, 2012 at 11:54 AM, Dominik Bruhn domi...@dbruhn.de wrote:
 Thanks Max for your answer.

 First, I do not understand your post. Why is it a problem if two of
 predictions match? From the formula for calculating R^2 I can see that
 there will be a DivByZero iff the total sum of squares is 0. This is
 only true if the predictions of all the predicted points from the
 test-set are equal to the mean of the test-set. Why should this happen?

 Anyway, I wrote the following code to check what you tried to tell:

 --
 library(caret)
 data(trees)
 formula=Volume~Girth+Height

 customSummary - function (data, lev = NULL, model = NULL) {
    print(summary(data$pred))
    return(defaultSummary(data, lev, model))
 }

 tc=trainControl(method='cv', summaryFunction=customSummary)
 train(formula, data=trees,  method='rpart', trControl=tc)
 --

 This outputs:
 ---
  Min. 1st Qu.  Median    Mean 3rd Qu.    Max.
  18.45   18.45   18.45   30.12   35.95   53.44
   Min. 1st Qu.  Median    Mean 3rd Qu.    Max.
  22.69   22.69   22.69   32.94   38.06   53.44
   Min. 1st Qu.  Median    Mean 3rd Qu.    Max.
  30.37   30.37   30.37   30.37   30.37   30.37
 [cut many values like this]
 Warning: In nominalTrainWorkflow(dat = trainData, info = trainInfo,
 method = method,  :
  There were missing values in resampled performance measures.
 -

 As I didn't understand your post, I don't know if this confirms your
 assumption.

 Thanks anyway,
 Dominik


 On 16/05/12 17:30, Max Kuhn wrote:
 More information is needed to be sure, but it is most likely that some
 of the resampled rpart models produce the same prediction for the
 hold-out samples (likely the result of no viable split being found).

 Almost 

Re: [R] Optimization problem

2012-05-17 Thread Pacin Al
Hi Greg,


The problem is that I also have restrictions for each variable (they must be
higher than -.07 and smaller than .2) and I'm dealing with a lot of them.

I've already tried the second approach but, as far as it seems, the function
doesn't satisfy my objective.  
That's what I'm doing:

.
faum = function(aum)
{
(...)
ifelse(colMeans(prob) .65, totm - (sum(marg)), totm - -1e10) 
#prob is a transformation of 'aum' and sum(marg) (always positive) is what I
want to maximize
(...)
return(-totm)
}

optim(rep(0,nrow), faum, method=L-BFGS-B, control=list(trace=12),
lower=rep(-.07,nrow), upper=rep(.2,nrow) )
.

The function gives me 1e10 if my initial values for the parameters doesn't
satisfy mean.65 and doesn't improve my initial conditions if they satisfy
mean=.65.

I know that a solution with mean.65 exists, I can find it by hand.

 

--
View this message in context: 
http://r.789695.n4.nabble.com/Optimization-problem-tp4630278p4630419.html
Sent from the R help mailing list archive at Nabble.com.

__
R-help@r-project.org mailing list
https://stat.ethz.ch/mailman/listinfo/r-help
PLEASE do read the posting guide http://www.R-project.org/posting-guide.html
and provide commented, minimal, self-contained, reproducible code.


Re: [R] caret: Error when using rpart and CV != LOOCV

2012-05-17 Thread Dominik Bruhn
Hy Max,
thanks again for the answer.

I checked the caret implementation and you were right. If the
predictions for the model constant (or sd(pred)==0) then the
implementation returns a NA for the rSquare (in postResample). This is
mainly because the caret implementation uses `cor` (from the
stats-package) which would throw a error for values with sd(pred)==0.

Do you know why this is implemented in this way? I wrote my own
summaryFunction which calculates rSquare by hand and it works fine. It
nevertheless does NOT(!) generate the same values as the original
implementation. It seems that the calcuation of Rsquare does not seem to
be consistent. I took mine from Wikipedia [1].

Here is my code:
---
customSummary - function (data, lev = NULL, model = NULL) {
 #Calulate rSquare
 ssTot - sum((data$obs-mean(data$obs))^2)
 ssErr - sum((data$obs-data$pred)^2)
 rSquare - 1-(ssErr/ssTot)

 #Calculate MSE
 mse - mean((data$pred - data$obs)^2)

 #Aggregate
 out - c(sqrt(mse), 1-(ssErr/ssTot))
 names(out) - c(RMSE, Rsquared)

 return(out)
}
---

[1]: http://en.wikipedia.org/wiki/Coefficient_of_determination#Definitions

Thanks!
Dominik




On 17/05/12 04:10, Max Kuhn wrote:
 Dominik,
 
 See this line:
 
   Min. 1st Qu.  MedianMean 3rd Qu.Max.
  30.37   30.37   30.37   30.37   30.37   30.37
 
 The variance of the predictions is zero. caret uses the formula for
 R^2 by calculating the correlation between the observed data and the
 predictions which uses sd(pred) which is zero. I believe that the same
 would occur with other formulas for R^2.
 
 Max
 
 On Wed, May 16, 2012 at 11:54 AM, Dominik Bruhn domi...@dbruhn.de wrote:
 Thanks Max for your answer.

 First, I do not understand your post. Why is it a problem if two of
 predictions match? From the formula for calculating R^2 I can see that
 there will be a DivByZero iff the total sum of squares is 0. This is
 only true if the predictions of all the predicted points from the
 test-set are equal to the mean of the test-set. Why should this happen?

 Anyway, I wrote the following code to check what you tried to tell:

 --
 library(caret)
 data(trees)
 formula=Volume~Girth+Height

 customSummary - function (data, lev = NULL, model = NULL) {
print(summary(data$pred))
return(defaultSummary(data, lev, model))
 }

 tc=trainControl(method='cv', summaryFunction=customSummary)
 train(formula, data=trees,  method='rpart', trControl=tc)
 --

 This outputs:
 ---
  Min. 1st Qu.  MedianMean 3rd Qu.Max.
  18.45   18.45   18.45   30.12   35.95   53.44
   Min. 1st Qu.  MedianMean 3rd Qu.Max.
  22.69   22.69   22.69   32.94   38.06   53.44
   Min. 1st Qu.  MedianMean 3rd Qu.Max.
  30.37   30.37   30.37   30.37   30.37   30.37
 [cut many values like this]
 Warning: In nominalTrainWorkflow(dat = trainData, info = trainInfo,
 method = method,  :
  There were missing values in resampled performance measures.
 -

 As I didn't understand your post, I don't know if this confirms your
 assumption.

 Thanks anyway,
 Dominik


 On 16/05/12 17:30, Max Kuhn wrote:
 More information is needed to be sure, but it is most likely that some
 of the resampled rpart models produce the same prediction for the
 hold-out samples (likely the result of no viable split being found).

 Almost every incarnation of R^2 requires the variance of the
 prediction. This particular failure mode would result in a divide by
 zero.

 Try using you own summary function (see ?trainControl) and put a
 print(summary(data$pred)) in there to verify my claim.

 Max

 On Wed, May 16, 2012 at 11:30 AM, Max Kuhn mxk...@gmail.com wrote:
 More information is needed to be sure, but it is most likely that some
 of the resampled rpart models produce the same prediction for the
 hold-out samples (likely the result of no viable split being found).

 Almost every incarnation of R^2 requires the variance of the
 prediction. This particular failure mode would result in a divide by
 zero.

 Try using you own summary function (see ?trainControl) and put a
 print(summary(data$pred)) in there to verify my claim.

 Max

 On Tue, May 15, 2012 at 5:55 AM, Dominik Bruhn domi...@dbruhn.de wrote:
 Hy,
 I got the following problem when trying to build a rpart model and using
 everything but LOOCV. Originally, I wanted to used k-fold partitioning,
 but every partitioning except LOOCV throws the following warning:

 
 Warning message: In nominalTrainWorkflow(dat = trainData, info =
 trainInfo, method = method, : There were missing values in resampled
 performance measures.
 -

 Below are some simplified testcases which repoduce the warning on my
 system.

 Question: What does this error mean? How can I avoid it?

 System-Information:
 -
 sessionInfo()
 R version 2.15.0 (2012-03-30)
 Platform: x86_64-pc-linux-gnu (64-bit)

 locale:
  [1] LC_CTYPE=en_GB.UTF-8   LC_NUMERIC=C
  [3] LC_TIME=en_GB.UTF-8

Re: [R] step function stops with subscript out of bounds

2012-05-17 Thread David A Vavra
Thanks.  

I guess I still don't understand what's going on. It's not at all intuitive
that the table used should be in the search path. Why is it searching for
the table? Isn't the table already stored in the model? If the documentation
says this, I haven't found it. Needing to know what names I can't use is a
bit disconcerting. I can think of names with high probability of uniqueness
but how could I ever be sure?

Table X is built on the fly in a script. What I supplied here was a
portion of the remaining script. I suppose the solution is to store it in an
environment in the search path. 

DAV


-Original Message-
From: Prof Brian Ripley [mailto:rip...@stats.ox.ac.uk] 
Sent: Thursday, May 17, 2012 6:46 AM
To: David A Vavra
Cc: r-help@r-project.org
Subject: Re: [R] step function stops with subscript out of bounds

On 17/05/2012 09:25, David A Vavra wrote:
 I've been having a problem using the step function to evaluate models.
I've
 simplified the code and get the same problem using the dataset Titanic.
The
 relevant code and output is below. The problem disappears (i.e., 'step'
runs
 correctly) if I rerun the code but change the 'loglm' call to explicitly
 reference Titanic instead of X (as in: loglm(as.formula(Y),data=Titanic)).

 What is causing this?

A lack of understanding of 'non-standard evaluation'.  X (or at least, 
the X you want) is not visible from the standard search path.


 TIA,
 DAV   

 --

 catn-function(...) cat(...,\n)
 local({  X-Titanic; print(class(X));
 Y-paste('~',paste(names(dimnames(X)),collapse=*));
 print(Y);
 sm-loglm(as.formula(Y),data=X);
 catn(SM); print(sm); catn('running');
 step(sm,direction='backward')  })

Which will tell you
Error in eval(expr, envir, enclos) : could not find function loglm

If you correct that and use a vanilla session you will get

Error in loglm(formula = ~Class + Sex + Age + Survived + Class:Sex + 
Class:Age +  :
   object 'X' not found

which is more informative.

So the solution is to

- use less easily masked names than 'X'.
- ensure the data object is visible on the search path.


 Output:

 [1] table
 [1] ~ Class*Sex*Age*Survived
 SM
 Call:
 loglm(formula = as.formula(Y), data = X)

 Statistics:
   X^2 df P(  X^2)
 Likelihood Ratio   0  01
 Pearson  NaN  01
 running
 Start:  AIC=64
 ~Class * Sex * Age * Survived

 Error in loglin(data, margins, start = start, fit = fitted, param = param,
 :
subscript out of bounds

 Enter a frame number, or 0 to exit

 1: local({
  X- Titanic
  print(class(X))
  Y- paste(~, paste(names(dimnames(X)), collapse = *))
  print(Y)
  sm- loglm(as.formula(Y), data = X
   2: eval.parent(substitute(eval(quote(expr), envir)))
   3: eval(expr, p)
   4: eval(expr, envir, enclos)
   5: eval(quote({
  X- Titanic
  print(class(X))
  Y- paste(~, paste(names(dimnames(X)), collapse = *))
  print(Y)
  sm- loglm(as.formula(Y), dat
   6: eval(expr, envir, enclos)
   7: #1: step(sm, direction = backward)
   8: #1: drop1(fit, scope$drop, scale = scale, trace = trace, k = k, ...)
   9: #1: drop1.default(fit, scope$drop, scale = scale, trace = trace, k =
k,
 ...)
 10: #1: update(object, as.formula(paste(~ . -, tt)), evaluate = FALSE)
 11: #1: update.loglm(object, as.formula(paste(~ . -, tt)), evaluate =
 FALSE)
 12: #1: eval.parent(call)
 13: #1: eval(expr, p)
 14: #1: eval(expr, envir, enclos)
 15: #1: loglm(formula = ~Class + Sex + Age + Survived + Class:Sex +
 Class:Age + Sex:Age + Class:Survived + Sex:Survived + Age:Survived +
 Class:Sex:Age + Class:
 16: #1: loglm1(formula, data, ..., .call = .call, .formula = .formula)
 17: #1: loglm1.default(formula, data, ..., .call = .call, .formula =
 .formula)
 18: #1: loglin(data, margins, start = start, fit = fitted, param = param,
 eps = eps, iter = iter, print = print)

 Selection: 0

 __
 R-help@r-project.org mailing list
 https://stat.ethz.ch/mailman/listinfo/r-help
 PLEASE do read the posting guide
http://www.R-project.org/posting-guide.html
 and provide commented, minimal, self-contained, reproducible code.


-- 
Brian D. Ripley,  rip...@stats.ox.ac.uk
Professor of Applied Statistics,  http://www.stats.ox.ac.uk/~ripley/
University of Oxford, Tel:  +44 1865 272861 (self)
1 South Parks Road, +44 1865 272866 (PA)
Oxford OX1 3TG, UKFax:  +44 1865 272595

__
R-help@r-project.org mailing list
https://stat.ethz.ch/mailman/listinfo/r-help
PLEASE do read the posting guide http://www.R-project.org/posting-guide.html
and provide commented, minimal, self-contained, reproducible code.


Re: [R] as.function parameters

2012-05-17 Thread Jason Edgecombe

On 05/17/2012 06:30 AM, jackl wrote:

Hi..

Ok here is an example on how I wanted the tree to be implemented in R:

-  the tree is, as you wrote, saved as a list of different tree levels
- each tree level is also saved as a list of different nodes in that
specific level
- and for the last part, each node is then saved as a list of functions

example:
tree- list(root, lvl1, lvl2)
root- list(node00)
lvl1- list(node10, node11)
lvl2- list(node20, node21, node22)

node00- list(f1,f2,f3)
node10- list(f1,f2,f3)
node11- list(f1,f2,f3)
..

note: I wrote f1, f2 and f3 in each node because it is the same function,
just with the different parameter, the stock price at that node.

I tried implementing a tree manually and I found out that the
independences between one node and each childnode cause a
heavy computation power.. (the function f3 contains f3 of the
two childnodes and so on..)
example:
node11$f3- max(node11$f2, node21$f3, node22$f3)

Im facing this problem even with a tree with 'only' 4-5 layers..

best thanks for any answers


Hi,

First, what problem are you trying to solve? A tree structure is not a 
problem... it's a tool, to solve a problem.


Second, a tree data structure is simply a simplified graph structure. 
There are many packages in R that deal with and store graph structures. 
Two that come to mind are igraph 
http://cran.r-project.org/web/packages/igraph/index.html and sna 
http://cran.r-project.org/web/packages/sna/


Since those packages can handle graphs, they can handle trees as well. 
As an added bonus, the code is already available and well-tested.


If memory usage is a concern, then look into using a sparse matrix 
implementation.


Jason

__
R-help@r-project.org mailing list
https://stat.ethz.ch/mailman/listinfo/r-help
PLEASE do read the posting guide http://www.R-project.org/posting-guide.html
and provide commented, minimal, self-contained, reproducible code.


[R] glm convergence warning

2012-05-17 Thread Sophie Baillargeon
Hi,

When I run the following code :

Y - c(rep(0,35),1,2,0,6,8,16,43)
cst - log(choose(42, 42:1)) 
beta - 42:1
tau - (beta^2)/2
fit - glm(formula = Y ~ offset(cst) + beta + tau, family = poisson)
fit
fit$converged

glm prints a warning saying that the algorithm did not converge.
However, fit$converged takes the value TRUE.

I don't understand why fit$converged is not always FALSE when the warning 
algorithm did not converge is produced. Could someone help me understand why 
I get this result?

Thanks a lot,
 
Sophie

__
R-help@r-project.org mailing list
https://stat.ethz.ch/mailman/listinfo/r-help
PLEASE do read the posting guide http://www.R-project.org/posting-guide.html
and provide commented, minimal, self-contained, reproducible code.


[R] solve the equation in R

2012-05-17 Thread Pei-Ling Lin
Hi all,

I need to solve this following equation which the unknown value (here is a) 
is on the exponent place, could anybody please help me to figure it out? I 
tried to use lm.sol() or solve() function but it doesn't work.. 

Thanks for help,
Peiling 


  (x,   y) = (2.21, 1.01)

  y = 1 + x - ( 1 + (x)^a)^(1/a)
  
 

__
R-help@r-project.org mailing list
https://stat.ethz.ch/mailman/listinfo/r-help
PLEASE do read the posting guide http://www.R-project.org/posting-guide.html
and provide commented, minimal, self-contained, reproducible code.


Re: [R] hmd.mx

2012-05-17 Thread krmartina
I used str(slovenia) and saw I was using wrong names - instead of year I
tried years and the same with age.

For hmd.mx you need package RCurl.

Thank you very much for your help!

Martina

--
View this message in context: 
http://r.789695.n4.nabble.com/hmd-mx-tp4630350p4630439.html
Sent from the R help mailing list archive at Nabble.com.

__
R-help@r-project.org mailing list
https://stat.ethz.ch/mailman/listinfo/r-help
PLEASE do read the posting guide http://www.R-project.org/posting-guide.html
and provide commented, minimal, self-contained, reproducible code.


Re: [R] solve the equation in R

2012-05-17 Thread R. Michael Weylandt
You could probably find a way to do this using nls(), optim() or
uniroot() but this seems to be very much a homework problem so I won't
say more.

solve() solves matrix linear equations... not what you want here. Read
the docs.

Michael

On Thu, May 17, 2012 at 2:53 PM, Pei-Ling Lin barthea...@gmail.com wrote:
 Hi all,

 I need to solve this following equation which the unknown value (here is a) 
 is on the exponent place, could anybody please help me to figure it out? I 
 tried to use lm.sol() or solve() function but it doesn't work..

 Thanks for help,
 Peiling


  (x,   y) = (2.21, 1.01)

  y = 1 + x - ( 1 + (x)^a)^(1/a)



 __
 R-help@r-project.org mailing list
 https://stat.ethz.ch/mailman/listinfo/r-help
 PLEASE do read the posting guide http://www.R-project.org/posting-guide.html
 and provide commented, minimal, self-contained, reproducible code.

__
R-help@r-project.org mailing list
https://stat.ethz.ch/mailman/listinfo/r-help
PLEASE do read the posting guide http://www.R-project.org/posting-guide.html
and provide commented, minimal, self-contained, reproducible code.


Re: [R] Windows Task Scheduler and R updates. Need basic tips

2012-05-17 Thread Joshua Wiley
Hi Paul,

This is a bit OT, but here's what I would do.

1) write the R script (or if updating packages is all you want Rscript
-e update.packages(repos = 'yourrepo', ask = FALSE) would do it
without need for a script)

2) write a silly batch file (e.g., Rupdate.bat) wrapper (maybe there
are better ways, but this is easy) that contains the command to have R
execute the R script (becomes slightly less silly if for example you
have multiple versions of R and would like to update all of them, then
the batch file could go through systemattically).  If you care, you
can redirect stderror and stdout from the batch file to any file you
choose.

3) With appropriate privileges run (untested):

  schtasks /Create /SC DAILY /TN Rupdate /TR c:/path/to/Rupdate.bat /ST 19:00

which would create a task Rupdate that would run daily at 19:00 and
execute Rupdate.bat.

Cheers,

Josh

On Thu, May 17, 2012 at 10:28 AM, Paul Johnson pauljoh...@gmail.com wrote:
 This is a basic Windows system administrator problem, asked by a Linux
 guy who is helping out in a Windows lab.

 I want to keep R packages up to date on MS Windows 7 with a job in the
 Task Scheduler.  I have an R program that I can run (as
 administrator) that updates the existing packages and then installs
 all the new ones.

 I do not understand how to run that in a dependable way in the scheduler.

 If I put the update script R-update.R in, for example, in

 C:\Program Files\R\R-update.R

 Then what?  Do I need a CMD batch script to run the R script?

 I can't tell where Windows wants to write the standard output and
 error for my R job.

 And while I'm asking, does Windows care if I run

 R CMD BATCH C:\Program Files\R\R-update.R

 or

 R --vanilla -f C:\Program Files\R\R-update.R

 pj
 --
 Paul E. Johnson
 Professor, Political Science    Assoc. Director
 1541 Lilac Lane, Room 504     Center for Research Methods
 University of Kansas               University of Kansas
 http://pj.freefaculty.org            http://quant.ku.edu

 __
 R-help@r-project.org mailing list
 https://stat.ethz.ch/mailman/listinfo/r-help
 PLEASE do read the posting guide http://www.R-project.org/posting-guide.html
 and provide commented, minimal, self-contained, reproducible code.



-- 
Joshua Wiley
Ph.D. Student, Health Psychology
Programmer Analyst II, Statistical Consulting Group
University of California, Los Angeles
https://joshuawiley.com/

__
R-help@r-project.org mailing list
https://stat.ethz.ch/mailman/listinfo/r-help
PLEASE do read the posting guide http://www.R-project.org/posting-guide.html
and provide commented, minimal, self-contained, reproducible code.


Re: [R] solve the equation in R

2012-05-17 Thread Petr Savicky
On Thu, May 17, 2012 at 01:53:39PM -0500, Pei-Ling Lin wrote:
 Hi all,
 
 I need to solve this following equation which the unknown value (here is a) 
 is on the exponent place, could anybody please help me to figure it out? I 
 tried to use lm.sol() or solve() function but it doesn't work.. 
 
 Thanks for help,
 Peiling 
 
 
   (x,   y) = (2.21, 1.01)
 
   y = 1 + x - ( 1 + (x)^a)^(1/a)

Hi.

First, plot the function

  f - function(a) { 1 + x - ( 1 + (x)^a)^(1/a) - y }
  x - 2.21
  y - 1.01
  a - seq(0.1, 20, length=1000)
  plot(a, f(a), ylim=c(-0.2, 0.2), type=l)
  abline(h=0)

This suggests, that there is no root. This may also be obtained
analytically.

Hope this helps.

Petr Savicky.

__
R-help@r-project.org mailing list
https://stat.ethz.ch/mailman/listinfo/r-help
PLEASE do read the posting guide http://www.R-project.org/posting-guide.html
and provide commented, minimal, self-contained, reproducible code.


Re: [R] solve the equation in R

2012-05-17 Thread Ted Harding
On 17-May-2012 21:50:17 Petr Savicky wrote:
 On Thu, May 17, 2012 at 01:53:39PM -0500, Pei-Ling Lin wrote:
 Hi all,
 
 I need to solve this following equation which the unknown value (here is
 a) is on the exponent place, could anybody please help me to figure it
 out? I tried to use lm.sol() or solve() function but it doesn't work.. 
 
 Thanks for help,
 Peiling 
 
 
   (x,   y) = (2.21, 1.01)
 
   y = 1 + x - ( 1 + (x)^a)^(1/a)
 
 Hi.
 
 First, plot the function
 
   f - function(a) { 1 + x - ( 1 + (x)^a)^(1/a) - y }
   x - 2.21
   y - 1.01
   a - seq(0.1, 20, length=1000)
   plot(a, f(a), ylim=c(-0.2, 0.2), type=l)
   abline(h=0)
 
 This suggests, that there is no root. This may also be obtained
 analytically.
 
 Hope this helps.
 
 Petr Savicky.

Well, certainly no solution for Y  1. But if y  1 then there
is a solution. Was the problem stated correctly?
Ted.

-
E-Mail: (Ted Harding) ted.hard...@wlandres.net
Date: 17-May-2012  Time: 23:27:09
This message was sent by XFMail

__
R-help@r-project.org mailing list
https://stat.ethz.ch/mailman/listinfo/r-help
PLEASE do read the posting guide http://www.R-project.org/posting-guide.html
and provide commented, minimal, self-contained, reproducible code.


Re: [R] nls and if statements

2012-05-17 Thread DWatts

Peter Ehlers wrote
 
 

 
 If the above data is all you have, I wouldn't chugg too long.
 Looking at plots of Cfl~Pw and Cfl~Tsoil says to me that you
 have far too much variability to fit your model.
 
 You might find this plot instructive:
 
library(rgl)
plot3d(Pw, Tsoil, Cfl, col=1+(Pw5), size=5)
 
 
 Peter Ehlers
 
 
I hadn't realized there was such a package, and wow, do I ever wish I'd
learned of that plot3d sooner!
I did have a substantially larger data set to work with, but after a much
more careful look I realized it would never fit the extra param to the low
Pw numbers-- the predictability just isn't there. The 3d plot confirms this,
of course. I did, however, get the help I was looking for, so thank you all!

Danielle

--
View this message in context: 
http://r.789695.n4.nabble.com/nls-and-if-statements-tp4630391p4630451.html
Sent from the R help mailing list archive at Nabble.com.

__
R-help@r-project.org mailing list
https://stat.ethz.ch/mailman/listinfo/r-help
PLEASE do read the posting guide http://www.R-project.org/posting-guide.html
and provide commented, minimal, self-contained, reproducible code.


[R] Optimization inconsistencies

2012-05-17 Thread Nathan Stephens
I have a very simple maximization problem where I'm solving for the vector
x:

objective function:
w'x = value to maximize

box constraints (for all elements of w):
low  x  high

equality constraint:
sum(x) = 1

But I get inconsistent results depending on what starting values I. I've
tried various packages but none seem to bee the very solver in Excel. Any
recommendations on what packages or functions I should try?

--Nathan

[[alternative HTML version deleted]]

__
R-help@r-project.org mailing list
https://stat.ethz.ch/mailman/listinfo/r-help
PLEASE do read the posting guide http://www.R-project.org/posting-guide.html
and provide commented, minimal, self-contained, reproducible code.


Re: [R] Optimization inconsistencies

2012-05-17 Thread peter dalgaard

On May 18, 2012, at 00:14 , Nathan Stephens wrote:

 I have a very simple maximization problem where I'm solving for the vector
 x:
 
 objective function:
 w'x = value to maximize
 
 box constraints (for all elements of w):
 low  x  high
 
 equality constraint:
 sum(x) = 1
 
 But I get inconsistent results depending on what starting values I. I've
 tried various packages but none seem to bee the very solver in Excel. Any
 recommendations on what packages or functions I should try?

Sounds like a linear programming problem, so perhaps one of the packages that 
are specialized for that? lpSolve looks like it should do it.

(As a general matter: There's nothing simple about constrained maximization 
problems, and generic optimizers aren't really geared towards dealing with 
large sets of constraints.)

 
 --Nathan
 
   [[alternative HTML version deleted]]
 
 __
 R-help@r-project.org mailing list
 https://stat.ethz.ch/mailman/listinfo/r-help
 PLEASE do read the posting guide http://www.R-project.org/posting-guide.html
 and provide commented, minimal, self-contained, reproducible code.

-- 
Peter Dalgaard, Professor,
Center for Statistics, Copenhagen Business School
Solbjerg Plads 3, 2000 Frederiksberg, Denmark
Phone: (+45)38153501
Email: pd@cbs.dk  Priv: pda...@gmail.com

__
R-help@r-project.org mailing list
https://stat.ethz.ch/mailman/listinfo/r-help
PLEASE do read the posting guide http://www.R-project.org/posting-guide.html
and provide commented, minimal, self-contained, reproducible code.


[R] New Eyes Needed to See Syntax Error

2012-05-17 Thread Rich Shepard

  One of many scripts to produce 4 lattice plots on one page keeps throwing
an error. I've tried manipulating the file to eliminate the error, but have
not been able to do so. The error is:


source('bicarb.R')

Error in source(bicarb.R) : bicarb.R:15:1: unexpected symbol
14: 15: hco33
   ^

  The 'h' is in column 0 so the caret would be column -1, but it's presented
as column 1. The file, bicarb.R is:

hco31 - qqmath(~ HCO3 | factor(basin), data = surfchem.cast, main = 
'Bicarbonate (Raw)',

  prepanel = prepanel.qqmathline,
  panel = function(x, ...) {
panel.qqmathline(x, ...)
panel.qqmath(x, ...)
  })

hco32 - qqmath(~ log10(HCO3 | factor(basin), data = surfchem.cast, main = 
'Bicarbonate (Log10)',

  prepanel = prepanel.qqmathline,
  panel = function(x, ...) {
panel.qqmathline(x, ...)
panel.qqmath(x, ...)
  })

hco33 - qqmath(~ sqrt(HCO3 | factor(basin), data = surfchem.cast, main = 
'Bicarbonate (Square Root)',

  prepanel = prepanel.qqmathline,
  panel = function(x, ...) {
panel.qqmathline(x, ...)
panel.qqmath(x, ...)
  })

hco34 - qqmath(~ HCO3^1/3 | factor(basin), data = surfchem.cast, main = 
'Bicarbonate (Cubic Root)',

  prepanel = prepanel.qqmathline,
  panel = function(x, ...) {
panel.qqmathline(x, ...)
panel.qqmath(x, ...)
  })

pdf('hco3.pdf')
plot(hco31, split = c(1,1,2,2), more = TRUE)
plot(hco32, split = c(1,1,2,2), more = TRUE) plot(hco33, split = c(1,2,2,2), 
more = TRUE)

plot(hco34, split = c(2,2,2,2), more = FALSE)
dev.off()

  If the error is not apparent to you, please suggest what I could try to
isolate the cause.

Rich

--
Richard B. Shepard, Ph.D.  |   Integrity - Credibility - Innovation
Applied Ecosystem Services, Inc.   |Helping Ensure Our Clients' Futures
http://www.appl-ecosys.com Voice: 503-667-4517  Fax: 503-667-8863

__
R-help@r-project.org mailing list
https://stat.ethz.ch/mailman/listinfo/r-help
PLEASE do read the posting guide http://www.R-project.org/posting-guide.html
and provide commented, minimal, self-contained, reproducible code.


Re: [R] New Eyes Needed to See Syntax Error

2012-05-17 Thread Rich Shepard

On Thu, 17 May 2012, Rich Shepard wrote:


Error in source(bicarb.R) : bicarb.R:15:1: unexpected symbol
14:
15: hco33
   ^


  These two lines were concatenated in the message on the mail list.

Rich

--
Richard B. Shepard, Ph.D.  |   Integrity - Credibility - Innovation
Applied Ecosystem Services, Inc.   |Helping Ensure Our Clients' Futures
http://www.appl-ecosys.com Voice: 503-667-4517  Fax: 503-667-8863

__
R-help@r-project.org mailing list
https://stat.ethz.ch/mailman/listinfo/r-help
PLEASE do read the posting guide http://www.R-project.org/posting-guide.html
and provide commented, minimal, self-contained, reproducible code.


Re: [R] New Eyes Needed to See Syntax Error

2012-05-17 Thread Mercier Eloi

On 12-05-17 04:34 PM, Rich Shepard wrote:
hco32 - qqmath(~ log10(HCO3 | factor(basin), data = surfchem.cast, 
main = 'Bicarbonate (Log10)',

  prepanel = prepanel.qqmathline,
  panel = function(x, ...) {
panel.qqmathline(x, ...)
panel.qqmath(x, ...)
  }) 

Missing a closing parenthesis after log10.
It should be :
log10(HCO3 | factor(basin))

Eloi

--
Eloi Mercier
Bioinformatics PhD Student, UBC
Paul Pavlidis Lab
2185 East Mall
University of British Columbia
Vancouver BC V6T1Z4

__
R-help@r-project.org mailing list
https://stat.ethz.ch/mailman/listinfo/r-help
PLEASE do read the posting guide http://www.R-project.org/posting-guide.html
and provide commented, minimal, self-contained, reproducible code.


Re: [R] New Eyes Needed to See Syntax Error

2012-05-17 Thread baptiste auguie
Your log10(HCO3 and sqrt(HCO3 seem to be missing closing brackets.

HTH,

baptiste

On 18 May 2012 11:34, Rich Shepard rshep...@appl-ecosys.com wrote:
  One of many scripts to produce 4 lattice plots on one page keeps throwing
 an error. I've tried manipulating the file to eliminate the error, but have
 not been able to do so. The error is:

 source('bicarb.R')

 Error in source(bicarb.R) : bicarb.R:15:1: unexpected symbol
 14: 15: hco33
   ^

  The 'h' is in column 0 so the caret would be column -1, but it's presented
 as column 1. The file, bicarb.R is:

 hco31 - qqmath(~ HCO3 | factor(basin), data = surfchem.cast, main =
 'Bicarbonate (Raw)',
  prepanel = prepanel.qqmathline,
  panel = function(x, ...) {
    panel.qqmathline(x, ...)
    panel.qqmath(x, ...)
  })

 hco32 - qqmath(~ log10(HCO3 | factor(basin), data = surfchem.cast, main =
 'Bicarbonate (Log10)',
  prepanel = prepanel.qqmathline,
  panel = function(x, ...) {
    panel.qqmathline(x, ...)
    panel.qqmath(x, ...)
  })

 hco33 - qqmath(~ sqrt(HCO3 | factor(basin), data = surfchem.cast, main =
 'Bicarbonate (Square Root)',
  prepanel = prepanel.qqmathline,
  panel = function(x, ...) {
    panel.qqmathline(x, ...)
    panel.qqmath(x, ...)
  })

 hco34 - qqmath(~ HCO3^1/3 | factor(basin), data = surfchem.cast, main =
 'Bicarbonate (Cubic Root)',
  prepanel = prepanel.qqmathline,
  panel = function(x, ...) {
    panel.qqmathline(x, ...)
    panel.qqmath(x, ...)
  })

 pdf('hco3.pdf')
 plot(hco31, split = c(1,1,2,2), more = TRUE)
 plot(hco32, split = c(1,1,2,2), more = TRUE) plot(hco33, split = c(1,2,2,2),
 more = TRUE)
 plot(hco34, split = c(2,2,2,2), more = FALSE)
 dev.off()

  If the error is not apparent to you, please suggest what I could try to
 isolate the cause.

 Rich

 --
 Richard B. Shepard, Ph.D.          |   Integrity - Credibility - Innovation
 Applied Ecosystem Services, Inc.   |    Helping Ensure Our Clients' Futures
 http://www.appl-ecosys.com     Voice: 503-667-4517      Fax: 503-667-8863

 __
 R-help@r-project.org mailing list
 https://stat.ethz.ch/mailman/listinfo/r-help
 PLEASE do read the posting guide http://www.R-project.org/posting-guide.html
 and provide commented, minimal, self-contained, reproducible code.

__
R-help@r-project.org mailing list
https://stat.ethz.ch/mailman/listinfo/r-help
PLEASE do read the posting guide http://www.R-project.org/posting-guide.html
and provide commented, minimal, self-contained, reproducible code.


Re: [R] New Eyes Needed to See Syntax Error

2012-05-17 Thread Rich Shepard

On Thu, 17 May 2012, Mercier Eloi wrote:


Missing a closing parenthesis after log10.


Eloi,

  A-ha! I knew new eyes would see what I kept missing.

Many thanks!

Rich

--
Richard B. Shepard, Ph.D.  |   Integrity - Credibility - Innovation
Applied Ecosystem Services, Inc.   |Helping Ensure Our Clients' Futures
http://www.appl-ecosys.com Voice: 503-667-4517  Fax: 503-667-8863

__
R-help@r-project.org mailing list
https://stat.ethz.ch/mailman/listinfo/r-help
PLEASE do read the posting guide http://www.R-project.org/posting-guide.html
and provide commented, minimal, self-contained, reproducible code.


Re: [R] glm convergence warning

2012-05-17 Thread Duncan Mackay

Hi Sophie

It helps if you do some detective work
Try

fit1 - glm(formula = Y ~ offset(cst) + beta + tau, family = 
poisson,trace = T, maxit = 200)


and compare

Regards

Duncan

Duncan Mackay
Department of Agronomy and Soil Science
University of New England
Armidale NSW 2351
Email: home: mac...@northnet.com.au

At 05:35 18/05/2012, you wrote:

Hi,

When I run the following code :

Y - c(rep(0,35),1,2,0,6,8,16,43)
cst - log(choose(42, 42:1))
beta - 42:1
tau - (beta^2)/2
fit - glm(formula = Y ~ offset(cst) + beta + tau, family = poisson)
fit
fit$converged

glm prints a warning saying that the algorithm did not converge.
However, fit$converged takes the value TRUE.

I don't understand why fit$converged is not always FALSE when the 
warning algorithm did not converge is produced. Could someone help 
me understand why I get this result?


Thanks a lot,

Sophie

__
R-help@r-project.org mailing list
https://stat.ethz.ch/mailman/listinfo/r-help
PLEASE do read the posting guide http://www.R-project.org/posting-guide.html
and provide commented, minimal, self-contained, reproducible code.


__
R-help@r-project.org mailing list
https://stat.ethz.ch/mailman/listinfo/r-help
PLEASE do read the posting guide http://www.R-project.org/posting-guide.html
and provide commented, minimal, self-contained, reproducible code.


[R] Integration of two dimension function

2012-05-17 Thread li li
Dear all,
   I have a function f - function(x,y,c){as.numeric(x*y  c) }
  I need to solve the value of c so that when I take the integral of
the function f from 0.2 to 0.8 with respect to x and from 0 to 1
with respect to y, the integral equal some prefixed value, say 0.025.
  It involves two dimension integration. Can anyone give some
help on this?
  Thank you so much in advance.
   Hannah

[[alternative HTML version deleted]]

__
R-help@r-project.org mailing list
https://stat.ethz.ch/mailman/listinfo/r-help
PLEASE do read the posting guide http://www.R-project.org/posting-guide.html
and provide commented, minimal, self-contained, reproducible code.


[R] Failure building any package

2012-05-17 Thread Noah Silverman
Hello,

I'm attempting to build a package using R 2.15.0 on OS X

I am getting a generic failure when performing a cran type check on the 
package.  Even with a very simple test package, it still fails int he same 
place.

Example:

In R:
rm(list=ls())
foo - function(x){print(x)}
package.skeleton(name=foo)

Then, at the command line:
R CMD build foo
R CMD check --as-cran foo_1.0.tar.gz 

Some output not included here
* checking whether package ‘foo’ can be installed … ERROR
Installation failed.
See ‘foo.Rcheck/00install.out’ for details.

Looking at the 00install.out file, I see:

* installing *source* package ‘foo’ ...
** R
Error in parse(outFile) : 5949:10: unexpected symbol
5948: c(01, 00, 00, 00, 01, 00, 00, 00, 00, 00, 00, 00, 00, 00, 
5949: e0, 3f
  ^
ERROR: unable to collate and parse R files for package ‘foo’


Can anyone help explain what is happening here?  This is the most generic and 
empty package I can think of, so not sure why a build is failing.

Thanks!


--
Noah Silverman
UCLA Department of Statistics
8117 Math Sciences Building
Los Angeles, CA 90095

__
R-help@r-project.org mailing list
https://stat.ethz.ch/mailman/listinfo/r-help
PLEASE do read the posting guide http://www.R-project.org/posting-guide.html
and provide commented, minimal, self-contained, reproducible code.


Re: [R] Failure building any package

2012-05-17 Thread steven mosher
Did you edit the description file and the namespace file and the  Rd files?

Although my tutorial is for windows if you follow steps 8 thru 10 on mac it
should work

http://stevemosher.wordpress.com/step-8-the-sample-package/



On Thu, May 17, 2012 at 7:44 PM, Noah Silverman noahsilver...@ucla.eduwrote:

 Hello,

 I'm attempting to build a package using R 2.15.0 on OS X

 I am getting a generic failure when performing a cran type check on the
 package.  Even with a very simple test package, it still fails int he same
 place.

 Example:

 In R:
 rm(list=ls())
 foo - function(x){print(x)}
 package.skeleton(name=foo)

 Then, at the command line:
 R CMD build foo
 R CMD check --as-cran foo_1.0.tar.gz

 Some output not included here
 * checking whether package ‘foo’ can be installed … ERROR
 Installation failed.
 See ‘foo.Rcheck/00install.out’ for details.

 Looking at the 00install.out file, I see:

 * installing *source* package ‘foo’ ...
 ** R
 Error in parse(outFile) : 5949:10: unexpected symbol
 5948: c(01, 00, 00, 00, 01, 00, 00, 00, 00, 00, 00, 00, 00, 00,
 5949: e0, 3f
  ^
 ERROR: unable to collate and parse R files for package ‘foo’


 Can anyone help explain what is happening here?  This is the most generic
 and empty package I can think of, so not sure why a build is failing.

 Thanks!


 --
 Noah Silverman
 UCLA Department of Statistics
 8117 Math Sciences Building
 Los Angeles, CA 90095

 __
 R-help@r-project.org mailing list
 https://stat.ethz.ch/mailman/listinfo/r-help
 PLEASE do read the posting guide
 http://www.R-project.org/posting-guide.html
 and provide commented, minimal, self-contained, reproducible code.


[[alternative HTML version deleted]]

__
R-help@r-project.org mailing list
https://stat.ethz.ch/mailman/listinfo/r-help
PLEASE do read the posting guide http://www.R-project.org/posting-guide.html
and provide commented, minimal, self-contained, reproducible code.


Re: [R] Failure building any package

2012-05-17 Thread Joshua Wiley
Hi Noah,

Can you put your package on github or at least upload the tar ball?
Although I agree with Steven that as is, the package will not pass
cran checks, that is not the error I would expect.

Some of the experts may have seen this before or know instantly, but
for everyone else, being able to try what you are trying may help.
Its also not a bad idea to have your code hosted somewhere anyway, so
you can slice two beets with one stroke as it were.

Cheers,

Josh

On Thu, May 17, 2012 at 7:44 PM, Noah Silverman noahsilver...@ucla.edu wrote:
 Hello,

 I'm attempting to build a package using R 2.15.0 on OS X

 I am getting a generic failure when performing a cran type check on the 
 package.  Even with a very simple test package, it still fails int he same 
 place.

 Example:

 In R:
 rm(list=ls())
 foo - function(x){print(x)}
 package.skeleton(name=foo)

 Then, at the command line:
 R CMD build foo
 R CMD check --as-cran foo_1.0.tar.gz

 Some output not included here
 * checking whether package ‘foo’ can be installed … ERROR
 Installation failed.
 See ‘foo.Rcheck/00install.out’ for details.

 Looking at the 00install.out file, I see:

 * installing *source* package ‘foo’ ...
 ** R
 Error in parse(outFile) : 5949:10: unexpected symbol
 5948:     c(01, 00, 00, 00, 01, 00, 00, 00, 00, 00, 00, 00, 00, 00,
 5949:     e0, 3f
              ^
 ERROR: unable to collate and parse R files for package ‘foo’


 Can anyone help explain what is happening here?  This is the most generic and 
 empty package I can think of, so not sure why a build is failing.

 Thanks!


 --
 Noah Silverman
 UCLA Department of Statistics
 8117 Math Sciences Building
 Los Angeles, CA 90095

 __
 R-help@r-project.org mailing list
 https://stat.ethz.ch/mailman/listinfo/r-help
 PLEASE do read the posting guide http://www.R-project.org/posting-guide.html
 and provide commented, minimal, self-contained, reproducible code.



-- 
Joshua Wiley
Ph.D. Student, Health Psychology
Programmer Analyst II, Statistical Consulting Group
University of California, Los Angeles
https://joshuawiley.com/

__
R-help@r-project.org mailing list
https://stat.ethz.ch/mailman/listinfo/r-help
PLEASE do read the posting guide http://www.R-project.org/posting-guide.html
and provide commented, minimal, self-contained, reproducible code.


[R] Financial Statements Date Subsetting

2012-05-17 Thread Brian Edmundson
Dear All,

I'm new at R, but I really just need a couple of things.  The first thing I
need is to figure out how to get each individual financial statement
(CF,BS,IS).  I need each individual one because getting them all at once
allows for formatting issues once it is a CSV.  The date subsetting is what
I need because I will be running a statistical model in excel.  I know I
could probably build a cleaner model in R, but I have already built it in
excel.

The first place I am stuck at is actually viewing the financials. I have
gotten it to work a few times, but i am trying to script the installation
sequence ( I still assume I have to load the packages as well - which I
have done).

I have tried this sequence downloading straight from the - i think - the
CRAN mirror and I have also tried downloading this sequence straight from
R-Project.com...

install.packages(TTR)
install.packages(zoo)
install.packages(Defaults)
install.packages(xts)
install.packages(quantmod)

this still (from both downloading sources) produces

viewFin(GE.f, BS, Q)
Error in viewFin(GE.f, BS, Q) : ‘x’ must be of type ‘financials’

Any help would be greatly appreciated - thank everyone so much!


-- 
**Collateral Control Analyst II
Banking Officer - BBT
200 East 2nd Street Winston Salem, NC
brianedmunds...@gmail.com

[[alternative HTML version deleted]]

__
R-help@r-project.org mailing list
https://stat.ethz.ch/mailman/listinfo/r-help
PLEASE do read the posting guide http://www.R-project.org/posting-guide.html
and provide commented, minimal, self-contained, reproducible code.


[R] Correlation in Rattle

2012-05-17 Thread avideh yesharim
Hi,

I recently installed Rattle for R 2.15.0 and all the functions work properly 
except for the Correlation button. After I choose the Correlation radio button 
in Explore tab, I click Execute, but nothing happens. No results at all. What 
is it that I am missing?

Thank you,
Avideh

[[alternative HTML version deleted]]

__
R-help@r-project.org mailing list
https://stat.ethz.ch/mailman/listinfo/r-help
PLEASE do read the posting guide http://www.R-project.org/posting-guide.html
and provide commented, minimal, self-contained, reproducible code.


Re: [R] Correlation Matrix

2012-05-17 Thread mahdi
thanks a lot dear. I will keep your advice in my mind.

--
View this message in context: 
http://r.789695.n4.nabble.com/Correlation-Matrix-tp4630389p4630448.html
Sent from the R help mailing list archive at Nabble.com.

__
R-help@r-project.org mailing list
https://stat.ethz.ch/mailman/listinfo/r-help
PLEASE do read the posting guide http://www.R-project.org/posting-guide.html
and provide commented, minimal, self-contained, reproducible code.


[R] Covariance matrix in R with non-numeric variables

2012-05-17 Thread nataraj
Dear R help forum members,

I am modeling a gaussian distribution for a computational biology application 
and I am working in the statistical package R. In this regard, my problem is 
that I have to construct a covariance matrix with variables (non-numeric) and 
the covariance matrix is to be used in an maximizers of the likelihood function 
to predict the variables in the matrix. I am unable to do that because I do not 
have an idea of how to construct a covariance matrix with non-numeric variable 
in the matrix.

Any help in this regard will be highly appreciated.

Thanks in advance.

Regards, B.Nataraj


[[alternative HTML version deleted]]

__
R-help@r-project.org mailing list
https://stat.ethz.ch/mailman/listinfo/r-help
PLEASE do read the posting guide http://www.R-project.org/posting-guide.html
and provide commented, minimal, self-contained, reproducible code.


[R] Proc AutoReg (SAS like Output in R)

2012-05-17 Thread anil
Hi,

I want to find out, how can i get a SAS like output for Proc Autoreg with
AR(1) and AR(2) terms.


Thanks...

--
View this message in context: 
http://r.789695.n4.nabble.com/Proc-AutoReg-SAS-like-Output-in-R-tp4630468.html
Sent from the R help mailing list archive at Nabble.com.

__
R-help@r-project.org mailing list
https://stat.ethz.ch/mailman/listinfo/r-help
PLEASE do read the posting guide http://www.R-project.org/posting-guide.html
and provide commented, minimal, self-contained, reproducible code.